Sunteți pe pagina 1din 128

Most Probable

Prelims Questions
Compilation of ClearIAS Daily MCQs

Set - 5

Sourced from the repository of questions used for the


ClearIAS Email Program

www.clearias.com
Top 5 Priority Items
You Should Learn
Before UPSC Prelims

1. C
learI A m
S Preli s Online Mock Test Series

2. C learIA CS Aurrent ffairs C


apsules

(Monthly M C Qs)

3. C
learI A S Most Probable Questions (Daily

M C
Qs)

4. Previous Year UPS C Questions – Practise

the UPS C Qns Re-Take Provided by C A


learI S

5. C learIA S Online Study Materials ( F EER )

Also don't miss:

C
learI AS Intelligent E m
li ination Techniques (I E
Ts)

www.clearias.com
MCQ Compilation- 5

1. With regard to Salt Satyagraha, consider the following


statements

1. It started from Sabarmati Ashram in Gujarat.


2. It was to protect the rights to make salt and export it to
outside India.
3. It led to the mass Civil Disobedience Movement.

Which among the above statements is/are correct?

(a) 1 only
(b) 2 and 3 only
(c) 1 and 3 only
(d) 1, 2 and 3

Ans: (c) 1 and 3 only

Learning Zone: On March 12, 1930, Mahatma Gandhi embarked


a historic Salt March from Sabarmati Ashram in Gujarat’s
Ahmedabad to the village of Dandi in the state’s coastal area to
protest against the steep tax the British levied on salt. The Salt
March began on March 12, 1930, and continued till April 6,
1930.

It was a 24-day Salt March, which was non-violent in nature, is


historically significant as it led to the mass Civil Disobedience
Movement.
3
Page
Why this question is important? National Salt Satyagraha
Memorial has been opened at Dandi in Navsari district, Gujarat.
It was dedicated to the nation on the occasion of Mahatma
Gandhi’s 71st death anniversary (30th January).

The memorial encapsulates the ideals of Mahatma Gandhi-


Agrah for Swadeshi, Swatchagrah and Satyagraha.

2. With regard to Polar Vortex, consider the following


statements

1. It is a whirling cone of high pressure over the poles.


2. It is strongest in winter months.
3. It spins in the stratosphere.

Which among the above statements is/are correct?

(a) 1 and 2 only


(b) 2 and 3 only
(c) 1 and 3 only
(d) 1, 2 and 3

Ans: (b) 2 and 3 only

Learning Zone: Polar Vortex is described as a whirling cone of


low pressure over the poles that is strongest in the winter months
due to the increased temperature contrast between the polar
regions and the mid-latitudes, such as the US and Europe.
The polar vortex spins in the stratosphere.
Usually, when the vortex is strongest, cold air is less-likely to
plunge deep into North America or Europe. In other words, it
4

forms a wall that protects the mid-latitudes from cold Arctic air.
Page
Why this question is important? Meteorologists have blamed a
phenomenon called the polar vortex for the bitter cold that has
descended on much of the central and eastern United States this
week, forcing residents to huddle indoors, closing schools and
businesses and cancelling flights.

3. With regard to the President’s address at the beginning of


the Budget Session, consider the following statements

1. It is a constitutional requirement.
2. It is drafted by the Cabinet and provides a broad framework
of the government’s agenda and direction.

Which among the above statements is/are correct?

(a) 1 only
(b) 2 only
(c) Both 1 and 2
(d) Neither 1 nor 2

Ans: (c) Both 1 and 2

Learning Zone: Article 87(1) says: “At the commencement of the


first session after each general election to the House of the People
and at the commencement of the first session of each year the
President shall address both Houses of Parliament assembled
together and inform Parliament of the causes of its summons.”

First Constitutional Amendment: Originally, the Constitution


required the President to address both Houses of Parliament at
5

the commencement of “every session”. This requirement was


Page

changed by the First Amendment to the Constitution.


Why this question is important? The President’s speech
essentially highlights the government’s policy priorities and plans
for the upcoming year. It is drafted by the Cabinet and provides a
broad framework of the government’s agenda and direction.

4. With regard to Nuclear Suppliers Group, consider the


following statements

1. It is mandatory that member countries should sign to the


Nuclear Non-Proliferation Treaty.
2. Its guidelines are binding on the parties.
3. It is popularly referred to as the “London Club”.

Which among the above statements is/are correct?


6

(a) 2 and 3 only


Page

(b) 1 only
(c) 3 only
(d) 1 and 2 only

Ans: (C) 3 only

Learning Zone:
● NSG is Brought in 1974– in response to the Indian nuclear
test (Smiling Buddha).
● It is a Multilateral export control regime.
● It is a Group of nuclear supplier countries that seek to
prevent nuclear proliferation by controlling the export of
materials, equipment and technology that can be used to
manufacture nuclear weapons.
● The NSG first met in November 1975 in London, and is thus
popularly referred to as the “London Club”.
● It is Not a formal organization, and its guidelines are not
binding. Decisions, including on membership, are made by
consensus.
● Membership: 48 supplier states.

Why this question is important? India has been seeking entry


into NSG, but China has repeatedly stonewalled its bid.
India is not a signatory to the NPT. China has stuck to its stand
that new members should sign the Nuclear Non-Proliferation
Treaty (NPT), making India’s entry difficult as the group is guided
by the consensus principle.

5. With regard to the Department of Industrial Policy and


Promotion (DIPP), consider the following statements
7
Page
1. It works under the ambit of the Ministry of Commerce and
Industry.
2. It looks into matters related to the promotion of internal
trade.

Which among the above statements is/are correct?

(a) 1 only
(b) 2 only
(c) Both 1 and 2
(d) Neither 1 nor 2

Ans: (c) Both 1 and 2

Learning Zone: It functions under the Ministry of Commerce and


Industry. It will look into matters related to the promotion of
internal trade, including retail trade, the welfare of traders and
their employees, facilitating ease of doing business and start-ups.
The matters related to internal trade were earlier under the
domain of the Ministry of Consumer Affairs.

Why this question is important? The Department of Industrial


Policy and Promotion (DIPP) has been renamed as the
Department for Promotion of Industry and Internal Trade.

6. With regard to Pradhan Mantri Shram-Yogi Maandhan


Yojana, consider the following statements

1. It is a scheme for unorganized sector workers.


2. It assures a monthly pension of Rs 3,000 from the age of 60
years.
8

3. Any person who crossed 30 years of age can join the


Page

scheme.
Which among the above statements is/are correct?

(a) 1 and 2 only


(b) 2 and 3 only
(c) 1 and 3 only
(d) 1, 2 and 3

Ans: (a) 1 and 2 only

Learning Zone: It is a scheme for the unorganised sector workers


with monthly income up to Rs 15,000. A sum of Rs 500 crore has
been allocated for the Scheme.
This scheme shall provide an assured monthly pension of Rs
3,000 from the age of 60 years on a monthly contribution of a
small affordable amount during their working age.
An unorganised sector worker joining pension yojana at the age
of 29 years will have to contribute only Rs 100 per month till the
age of 60 years. A worker joining the pension yojana at 18 years,
will have to contribute as little as Rs 55 per month only.
The Government will deposit an equal matching share in the
pension account of the worker every month

Why this question is important? It is expected that at least 10


crore labourers and workers in the unorganised sector will avail
the benefit of the scheme within the next five years making it one
of the largest pension schemes of the world.

7. With regard to Denotified and Nomadic Tribes, consider


9

the following statements


Page
1. They were listed or notified as ‘born criminal ‘by the British
under a number of laws.
2. After India gained Independence, these tribes were ‘de-
notified’ from the list of Criminal Tribes.

Which among the above statements is/are correct?

(a) 1 only
(b) 2 only
(c) Both 1 and 2
(d) Neither 1 nor 2

Ans: (c) Both 1 and 2

Learning Zone: The Denotified Tribes are communities that were


listed or notified as ‘born criminal ‘by the British under a number
of laws. The term, ‘Denotified and Nomadic Tribes’, can be traced
to the Criminal Tribes Act (CTA) of 1871.

The colonial government notified nearly 200 tribal communities


to be hereditary criminals, cementing their social identity as
outcasts and subjecting them to constant harassment by the
administration.
Forest laws that came into force from the mid-nineteenth century
onwards deprived a large number of communities of their
traditional rights of hunting and gathering. The new laws
criminalised their very source of livelihood when it practised.
When the forests were cleared by the British for commercial use
and forest communities asked to contribute to labour, some
communities resisted and were declared ‘criminal’.
10
Page
The British thought that communities had lost their legitimate
means of livelihood, they must have been living by indulging in

criminal activities due to the arrival of road and railway


ilway
networks. After India gained Independence, these tribes were ‘de
‘de-
notified’ from the list of Criminal Tribes.
11
Page
Why this question is important? The Centre will form a welfare
panel for nomadic, semi-nomadic and de-notified communities.
Announced in Budget.

A Welfare Development Board will also be set up under the


Ministry of Social Justice and Empowerment to design and
implement programmes for these hard-to-reach communities.

8. Which among the following refers to INSTEX, recently


seen in the news?

(a) It is a payment mechanism being set up by the European


Union
(b) It is a skill development program initiated by MHRD
(c) Mobile application developed by Ministry of Textiles
(d) Award for technological innovation by UN

Ans: (a) It is a payment mechanism being set up by the


European Union

Learning Zone: It is a payment mechanism being set up by the


European Union to secure trade with Iran and skirt US sanctions

after Washington pulled out of the landmark nuclear deal last


May.

Why this question is important? This mechanism is the first


concrete step by the EU to counter Trump’s unilateral decision to
12

withdraw from the nuclear deal.


Page
9. Which among the following parameters are used to assess
a bank under Prompt Corrective Action (PCA) by RBI?

1. Capital ratios
2. Asset Quality
3. Profitability

Select the correct answer using the codes given below:

(a) 1 and 2 only


(b) 2 and 3 only
(c) 1 and 3 only
(d) 1, 2 and 3

Ans: (d) 1,2 and 3

Learning Zone: PCA norms allow the regulator to place certain


restrictions such as halting branch expansion and stopping
dividend payment. It can even cap a bank’s lending limit to one
entity or sector. Other corrective actions that can be imposed on
banks include special audit, restructuring operations and
activation of recovery plan. Banks’ promoters can be asked to
bring in new management, too. The RBI can also supersede the
bank’s board, under PCA.
The PCA is invoked when certain risk thresholds are breached.
There are three risk thresholds which are based on certain levels
of asset quality, profitability, capital and the like. The third such
threshold, which is the maximum tolerance limit, sets net NPA at
over 12% and a negative return on assets for four consecutive
years.
13
Page
Why this question is important? The Reserve Bank of India
(RBI) has lifted the Prompt Corrective Action (PCA) framework
operational curbs on Bank of India (BoI), Bank of Maharashtra
(BoM) and Oriental Bank of Commerce (OBC).

The PCA restrictions were lifted after these banks provided a


written commitment that they would comply with the norms of
minimum regulatory capital, net NPAs (Non-performing Assets)
and leverage ratio on an ongoing basis. These Banks have also
apprised RBI of the structural and systemic improvements they
have put in place.

10. Which among the following is the major objective of


Pradhan Mantri Kisan Samman Nidhi?

(a) To provide insurance to the crops of marginal farmers.


(b) To provide MSP to all the Kharif crops.
(c) To provide assured income support to the small and
marginal farmers.
(d) To waive all the agriculture loans up to 2018.

Ans: (c) To provide assured income support to the small and


marginal farmers.

Learning Zone: To provide assured income support to the small


and marginal farmers, the Government is launching the Pradhan
Mantri Kisan Samman Nidhi (PM-KISAN).
14
Page
Why this question is important? Under this programme,
vulnerable landholding farmer families, having cultivable land up
to 2 hectares, will be provided direct income support at the rate
of Rs. 6,000 per year.

This income support will be transferred directly into the bank


accounts of beneficiary farmers, in three equal instalments of Rs.
2,000 each.

11. With regard to New e-commerce policy, consider the


following statements

1. It disallows e-commerce players to control the inventory of


the vendors.
2. It bars entering into exclusive deals with brands for selling
products only on online platforms.

Which among the following statements is/are correct?

(a) 1 only
(b) 2 only
(c) Both 1 and 2
(d) Neither 1 nor 2

Ans: (c) Both 1 and 2

Learning Zone:
● It Bars online retailers from selling products through
vendors in which they have an equity interest.
● Also bars them from entering into exclusive deals with
brands for selling products only on their platforms.
15

● All online retailers will be required to maintain a level


Page

playing field for all the vendors selling their products on the
platform, and it shall not affect the sale prices of goods in
any manner.
● Disallows e-commerce players to control the inventory of the
vendors. Any such ownership over the inventory will convert
it into inventory based model from marketplace based
model, which is not entitled to FDI.
● Under the new rules, the e-commerce retailer shall be
deemed to own the inventory of a vendor if over 25 per cent
of the purchases of such a vendor is through it.

Why this question is important? The key objective behind the


revising the FDI rules for the e-commerce giants is to level the
playing field in the retail space, as heavy discounting on online
retail sites was causing heavy losses to the small and medium
brick and mortar stores.

12. Which among the following is not an objective of


Rashtriya Gokul Mission?

1. To get foreign breeds to produce more milk.


2. To raise the quality of Indian cows and bulls.
3. To outdo Jerseys and Holsteins.

Select the correct answer using the codes given below:

(a) 1 and 3 only


(b) 2 and 3 only
(c) 1 and 2 only
(d) 1, 2 and 3

Ans: (b) 2 and 3 only


16
Page
Learning Zone: To conserve and develop indigenous bovine
breeds, the government has launched ‘Rashtriya Gokul Mission’
under the National Programme for Bovine Breeding and Dairy
Development (NPBBD).

Why this question is important? The Mission is being


implemented with the objectives to a) development and
conservation of indigenous breeds b) undertake breed
improvement programme for indigenous cattle breeds so as to
improve the genetic makeup and increase the stock; c) enhance
milk production and productivity; d) upgrade nondescript cattle
using elite indigenous breeds like Gir, Sahiwal, Rathi, Deoni,
Tharparkar, Red Sindhi and e) distribute disease free high
genetic merit bulls for natural service.

13. Which among the following statements is/are true


regarding National Grid?

1. All states and Union territories are connected to the


National Grid.
2. The National Grid is owned, operated, and maintained by
state-owned Power Grid Corporation of India.
3. It will benefit from better availability resulting in lesser
power cuts.

Select the correct answer using the codes given below:

(a) 1 and 2 only


(b) 2 and 3 only
(c) 1, 2 and 3
(d) 1 and 3 only
17
Page

Ans: (b) 2 and 3 only


Learning Zone: It is the high-voltage electric power transmission
network in mainland India, connecting power stations and major
substations and ensuring that electricity generated anywhere in
mainland India can be used to satisfy demand elsewhere.

The National Grid is owned, operated, and maintained by state-


owned Power Grid Corporation of India.
It is one of the largest operating synchronous grids in the world
with 307.8 GW of installed power generation capacity.
The union territories of Andaman and Nicobar Islands and
Lakshadweep are not connected to the National Grid.

Why this question is important? PM Modi recently dedicated


the 220 kV Srinagar- Alusteng – Drass- Kargil – Leh
Transmission Line to the Nation. With this, Ladakh is now
connected to the National Grid.

14. Know My India Programme, often seen in the news, is


initiated by which among the following?

(a) Art of Living Foundation


(b) National Foundation for Communal Harmony (NFCH)
(c) Ministry of Human Resources And Development
(d) Ministry of Home

Ans: (b) National Foundation for Communal Harmony (NFCH)

Learning Zone: NFCH is an autonomous organization under the


administrative control of the Ministry of Home Affairs. The main
18

objective of the Foundation is to provide assistance to the


Page

children/youth rendered orphan/destitute in communal, caste,


ethnic or terrorist violence for their rehabilitation besides
promoting communal harmony and national integration through
various activities.

Why this question is important? It is a unique programme


initiated by the NFCH to bring together financially assisted
children of the Foundation from different States/Regions of the
country to promote oneness, fraternity and national integration.

15. With regard to Bharat Rang Mahotsav (BRM), consider the


following statements

1. It is the annual international theatre festival of India


2. It is organized by the Ministry of Culture.

Which among the above statements is/are correct?

(a) 1 only
(b) 2 only
(c) Both 1 and 2
(d) Neither 1 nor 2

Ans: (b) 2 only

Learning Zone:
Bharat Rang Mahotsav (BRM): It is the annual international
theatre festival of India organized by the National School of
Drama (NSD).
It was established two decades ago to stimulate the growth and
development of theatre across India.
Originally a national festival showcasing the work of the most
19

creative theatre workers in India, BRM has evolved to


Page
international scope, hosting theatre groups from around the
world, and is now the largest theatre festival of Asia.

Why this question is important? 20th edition of Bharat Rang


Mahotsav is being held in New Delhi.

16. With regard to the National Sports Development Fund,


consider the following statements

1. It supports sportspersons to excel in the field by providing


opportunities to train under coaches of international repute.
2. The Fund is managed by a Sports Authority of India.

Which among the above statements is/are correct?

(a) 1 and 2 only


(b) 2 and 3 only
(c) 1 and 3 only
(d) 1, 2 and 3

Ans: (a) 1 only

Learning Zone: Established in 1998, under the Charitable


Endowments Act 1890, vide Government of India Notification
dated 12th November 1998.

Functions:
The NSDF supports sportspersons to excel in the field by
providing opportunities to train under coaches of international
repute with technical, scientific and psychological support and
20

also in getting exposure to international competitions.


Page
Financial assistance is also provided to specific projects for the
promotion of sports and games sponsored by reputed
Organizations/Institutes, provided the facilities so created are
made available to a sizeable population of the area/region.
The Fund is managed by a Council constituted by the Central
Government.
Union Minister for Youth Affairs and Sports is the Chairperson of
the council.

Why this question is important? The Ministry of Youth Affairs


& Sports is implementing Target Olympic Podium Scheme (TOPS)
within the overall ambit of National Sports Development Fund
(NSDF) for providing financial assistance to elite athletes
included in TOPS for their customized training in world-class
training institutes/academies within the country and abroad.

17. With regard to Draft National River Ganga Bill, 2018,


consider the following statements

1. The bill proposes to ban the construction of jetties and ports


on Ganga.
2. It defines Ganga as ‘India’s National River’.
3. Armed Ganga Protection Corps (GPC) will have the power to
arrest those who pollute the river covering offences like
obstructing the flow of the river.

Which among the above statements is/are correct?

(a) 1 only
(b) 1 and 3 only
(c) 2 only
21

(d) 1, 2 and 3
Page
Ans: (d) 1,2 and 3

Learning Zone: The bill proposes to ban the construction of


jetties, ports or “permanent hydraulic structures” in the Ganga
unless permitted by the National Ganga Rejuvenation Authority.
It proposes to create a management structure that will supervise
the health of the 2,500-kilometre-long Ganga which, the draft Bill
defines, as ‘India’s national river.’
The Bill lays down a host of restrictions to ensure the
“uninterrupted, ecological flow” of the river. Currently, a host of
dams in the upper stretches of the river lead to the river’s flow
being obstructed.
The Armed Ganga Protection Corps (GPC) personnel will be
provided by the ministry of home affairs and will be deployed by
the National Ganga Rejuvenation Authority. The GPC personnel
will have the power to arrest those who pollute the river covering
offences like obstructing the flow of the river to commercial
fishing.

Why this question is important? According to a map of Ganga


river water quality presented by the Central Pollution Control
Board (CPCB) to National Green Tribunal (NGT) in August 2018,
only five out of 70-odd monitoring stations had water that was fit
for drinking and seven for bathing. After three decades of efforts
to clean the national river, it is a sad state of affairs that the river
is not even fit for bathing.

18. With regard to the Pre-Departure Orientation Programme,


consider the following statements
22
Page

1. It is an initiative by the Ministry of External Affairs.


2. It is a mandatory program to all who seeks a job outside
India.

Which among the above statements is/are correct?

(a) 1 only
(b) 2 only
(c) Both 1 and 2
(d) Neither 1 nor 2

Ans: (a) 1 only

Learning Zone: The Ministry of External Affairs (MEA) in


collaboration with Ministry of Skill Development and
Entrepreneurship (MSDE) is conducting the PDOT programme
under the Pravasi Kaushal Vikas Yojana (PKVY).
The National Skill Development Corporation is the implementing
agency for this programme.

Why this question is important? Given the need to orient


potential migrant workers with regard to language, culture, do’s
and don’ts in the destination country, the emigration process and
welfare measures a Pre-Departure Orientation Training (PDOT)
programme have been launched.

19. With regard to the Indian Ocean Rim Association (IORA),


consider the following statements

1. Its objective is sustainable development within the Indian


Ocean Region.
2. All the Indian Ocean surrounding countries are a party to it.
23
Page

Which among the above statements is/are correct?


(a) 1 only
(b) 2 only
(c) Both 1 and 2
(d) Neither 1 nor 2

Ans: (a) 1 only

Learning Zone: The Indian Ocean Rim Association was set up


with the objective of strengthening regional cooperation and
sustainable development within the Indian Ocean Region
The IORA is a regional forum, tripartite in nature, bringing
together representatives of Government, Business and Academia,
for promoting co-operation and closer interaction among them.
India, Australia, Iran, Indonesia, Thailand, Malaysia, South
Africa, Mozambique, Kenya, Sri Lanka, Tanzania, Bangladesh,
Singapore, Mauritius, Madagascar, UAE, Yemen, Seychelles,
Somalia, Comoros and Oman are among the members of IORA.

Why this question is important? The Ministry of Home Affairs


in collaboration with the Ministry of External Affairs and National
Disaster Management Authority (NDMA) is organising a meeting
of Indian Ocean Rim Association (IORA) Cluster Group on
Disaster Risk Management (DRM) in New Delhi.

20. Which among the following the purpose of National


Testing Agency?

(a) To conduct safe nuclear tests


(b) To conduct agricultural experiments for drought-resistant
24

crop
Page
(c) To conduct entrance examinations for Higher Education
Institutions (HEIs).
(d) To conduct and evaluate the efficacy of patented drugs.

Ans: (c) To conduct entrance examinations for Higher


Education Institutions (HEIs).

Learning Zone: In pursuance of the Budget Announcement


2017-18, the Union Cabinet, in November 2017, approved the
creation of the National Testing Agency (NTA) as an autonomous
and self-sustained premier testing organization to conduct
entrance examinations for Higher Education Institutions (HEIs)
in the country.

Composition: It will be chaired by an educationist who will be


appointed by the MHRD. The agency will have a board of
governors who will represent the member institutions.

Why this question is important? National Testing Agency (NTA)


has launched a ‘mobile app’ through which students can practice
or take mock tests on their own computers or smartphones.

21. With regard to Rashtriya Rashtriya Vayoshri Yojana


(RVY), consider the following statements

1. The scheme is for all senior citizens in the country.


2. It is initiated by the Ministry of Health and Family Welfare.

Which among the above statements is/are correct?

(a) 1 only
25

(b) 2 only
Page

(c) Both 1 and 2


(d) Neither 1 nor 2

Ans: (d) Neither 1 nor 2

Learning Zone: The Scheme aims at providing Senior Citizens,


belonging to BPL category and suffering from any of the age-
related disability/infirmity Low vision, Hearing impairment, Loss
of teeth and Locomotor disability, with such assisted-living
devices which can restore near normalcy in their bodily
functions, overcoming the disability/infirmity manifested.

Why this question is important? This is a Central Sector


Scheme, fully funded by the Central Government. The
expenditure for implementation of the scheme will be met from
the “Senior Citizens’ Welfare Fund“.
Under the scheme, free of cost distribution of the devices,
commensurate with the extent of disability/infirmity that is
manifested among the eligible senior citizens will take place.

22. With regard to the World LPG Association, consider the


following statements?

1. It supports the development of LPG markets.


2. Asia LPG Summit recently held in New Delhi is organized by
the World LPG Association.

Which among the above statements is/are correct?

(a) 1 only
(b) 2 only
(c) Both 1 and 2
26

(d) Neither 1 nor 2


Page
Ans: (c) Both 1 and 2

Learning Zone: The World LPG Association (WLPGA) is the


authoritative global voice for the liquefied petroleum gas (LPG)
industry and the worldwide industry association which
represents the interests of the LPG industry globally.

The WLPGA promotes the use of LPG to foster a cleaner, healthier


and more prosperous world.

The Association's mission is to:


Demonstrate the benefits of LPG and inform, educate and
influence all stakeholders.
Support the development of LPG markets.
Promote compliance with standards, good business and good
safety practices.
Identify innovation and facilitate knowledge transfer.

Why this question is important? The 2nd edition of the Asia


LPG Summit is being held in New Delhi.
The summit is being organized by the World LPG Association
(WLPGA) in association with major Indian public-sector oil
companies (OMCs) Indian Oil, Hindustan Petroleum, and Bharat
Petroleum.

23. With regard to the Safe City project, consider the


following statements

1. It is to be implemented in all cities in the country.


2. The project would be implemented as a Centrally Sponsored
27

Scheme.
Page
3. It aims to Set up Women Help Desks in all Police Stations
with Counsellors.

Which among the above statements is/are correct?

(a) 1 and 2 only


(b) 2 and 3 only
(c) 1 and 3 only
(d) 1, 2 and 3

Ans: (d) 1, 2 and 3

Learning Zone: Empowered Committee of Officers for Nirbhaya


funds scheme has approved pilot Safe City projects in eight
selected metropolitan cities, namely, Delhi, Kolkata, Mumbai,
Chennai, Bengaluru, Hyderabad, Ahmedabad and Lucknow with
a view to providing safety to women in public places.

The project would be implemented as a Centrally Sponsored


Scheme with Centre and State sharing the funding in 60:40 ratio.
The major components of the Safe City project include inter-alia:

Setting up an Integrated Smart Control Room;


Setting up Pink Out-posts (exclusively administered by Women
police) for facilitating ease of filing complaint by women;
Pink Patrols of Women police;
Setting up Women Help Desks in all Police Stations with
Counsellors;
Augmentation of existing Asha Jyoti Kendra;
Implementing Safety measures in buses, including Cameras;
Improving Street Lighting in identified Hot Spot areas;
28

Setting up Pink Toilets;


Page
Integration of Women power-help
power line with a single Emergency
number.

24. With regard to the Kisan Credit Card (KCC) scheme,


29

consider the following statements


1. It was announced in the Budget speech of 2015
2015-2016.
Page
2. The KCC scheme is being implemented by the all Co-
operative banks, Regional Rural Banks and Public Sector
Banks throughout the country.
3. The scheme covers the risk of KCC holders against death
only.

Which among the above statements is/are correct?

(a) 1 and 3 only


(b) 2 and 3 only
(c) 2 only
(d) 1 only

Ans: (c) 2 only

Learning Zone: The Kisan Credit Card (KCC) scheme was


announced in the Budget speech of 1998-99 to fulfil the financial
requirements of the farmers at various stages of farming through
institutional credit.
The model scheme was prepared by the National Bank for
Agriculture and Rural Development (NABARD) on the
recommendation of the V Gupta committee.
The KCC scheme is being implemented by the all Co-operative
banks, Regional Rural Banks and Public Sector Banks
throughout the country.
The scheme covers the risk of KCC holders against death or
permanent disability resulting from accidents.

Why this question is important? The Indian Banking


Association (IBA) has issued advisory guidelines requesting
30

banks to waive off the processing, documentation, inspection,


Page

ledger folio charges and all other service charges for crop loans
up to Rs 3 Lakhs or for the Kisan Credit Card Scheme. The move
aims to provide direct benefit to farmers and ease the pressure on
them.

25. With regard to Banning of Unregulated Deposit Schemes


Bill, 2018, consider the following statements

1. It aims to create an online central database, for collection


and sharing of information on deposit-taking activities in
the country.
2. It bans Deposit Takers from promoting, operating, issuing
advertisements or accepting deposits in any Unregulated
Deposit Scheme.

Which among the above statements is/are correct?

(a) 1 only
(b) 2 only
(c) Both 1 and 2
(d) Neither 1 nor 2

Ans: (c) Both 1 and 2

Learning Zone: Key provisions of the Bill:

Substantive banning clause which bans Deposit Takers from


promoting, operating, issuing advertisements or accepting
deposits in any Unregulated Deposit Scheme. The Bill bans
unregulated deposit-taking activities altogether, by making them
31

an offence ex-ante rather than the existing legislative-cum-


Page
regulatory framework which only comes into effect ex-post with
considerable time lags.
Creation of three different types of offences, namely, running of
Unregulated Deposit Schemes, fraudulent default in Regulated
Deposit Schemes, and wrongful inducement in relation to
Unregulated Deposit Schemes.
Severe punishment and heavy pecuniary fines to act as a
deterrent.
Provisions for disgorgement or repayment of deposits in cases
where such schemes nonetheless manage to raise deposits
illegally.
Attachment of properties/assets by the Competent Authority, and
subsequent realization of assets for repayment to depositors.
Clear-cut timelines have been provided for attachment of
property and restitution to depositors.
Creation of an online central database, for collection and sharing
of information on deposit-taking activities in the country.

Why this question is important? To deal with the menace of


illicit deposit-taking schemes, as in the recent past, there have
been rising instances of people in various parts of the country
being defrauded by illicit deposit-taking schemes.

The worst victims of these schemes are the poor and the
financially illiterate, and the operations of such schemes are
often spread over many States.

26. With regard to Income-Tax Ombudsman, consider the


following statements
32

1. It was established for resolution of complaints relating to


Page

the Income Tax Department like an overdue refund.


2. It was autonomous and hence independent of the
jurisdiction of the Income-tax department.
3. Its operations are largely effective in achieving its objectives.

Which among the above statements is/are correct?

(a) 1 and 2 only


(b) 2 and 3 only
(c) 1 and 3 only
(d) 1, 2 and 3

Ans: (a) 1 and 2 only

Learning Zone: The Institution of Income-Tax Ombudsman was


created in the year 2003 to deal with grievances of public related
to the settlement of complaints relating to Income Tax. However,
the Institution of Ombudsman failed to achieve its objectives.

Why this question important? The Union Cabinet chaired has


approved the proposal for Abolition of Institution of Income-Tax
Ombudsman and Indirect Tax Ombudsman.

27. With regard to Agri-Market Infrastructure Fund (AMIF),


consider the following statements

1. It will be created with the Rural Development Bank for


development and up-gradation of agricultural marketing
infrastructure.
2. AMIF will provide the State/UT Governments subsidized
33

loan for their proposal for developing marketing


Page

infrastructure.
Which among the above statements is/are correct?

(a) 1 only
(b) 2 only
(c) Both 1 and 2
(d) Neither 1 nor 2

Ans: (b) 2 only

Learning Zone: The fund will be created with NABARD for


development and up-gradation of agricultural marketing
infrastructure in Gramin Agricultural Markets and Regulated
Wholesale Markets.
AMIF will provide the State/UT Governments subsidized loan for
their proposal for developing marketing infrastructure in 585
Agriculture Produce Market Committees (APMCs) and 10,000
Grameen Agricultural Markets (GrAMs).
States may also access AMIF for innovative integrated market
infrastructure projects including Hub and Spoke mode and in
Public Private Partnership mode.

Why this question is important? The Cabinet Committee on


Economic Affairs has given its approval for the creation of a
corpus of Rs. 2000 crore for Agri-Market Infrastructure Fund
(AMIF).

28. ‘Darwaza Band’ campaign, often seen in news is related


to:
34

(a) Women’s safety


Page

(b) Open defecation


(c) Science education
(d) Child Safety

Ans: (b) Open defecation

Learning Zone: The campaign, produced by the Ministry of


Drinking Water and Sanitation, and supported by the World
Bank was launched in Mumbai.
The campaign talks about how a toilet must be used by all,
always and under all circumstances (har koi, har roz, hamesha).
The focuses on sustaining the open defecation free status of
villages across the country.
It focuses on ensuring that people’s behaviour is changed for
good and everyone always uses a toilet.

Why this question is important? The Swachh Bharat Mission


Grameen has launched the ‘Darwaza Band -Part 2’ campaign
starring Amitabh Bachchan.

29. ‘Zero Fatality Corridor’ initiative often seen in the news,


is an initiative of which among the following?

(a) Ministry of Road Transport and Highways.


(b) Save LIFE Foundation
(c) Amnesty International
(d) Govt. of Maharashtra

Ans: (b) Save LIFE Foundation


35

Learning Zone: The Save LIFE Foundation’s ZFC model is well-


Page

known for reducing road crash deaths by 30 per cent on the


Mumbai-Pune Expressway. It attempts to reduce road crash
deaths through a 360-degree intervention, panning road
engineering, police enforcement, rapid emergency care and road-
user engagement.

Why this question is important? The Delhi government has


kicked off its pilot project of creating Delhi’s first ‘zero fatality
corridor’. It has also issued actionable points to all agencies to
ensure a 10% reduction in road accidents by the end of the year.

30. With regard to the Voter Verifiable Paper Audit Trail


(VVPAT) system, consider the following statements

1. There shall be 100% use of the Voter Verifiable Paper Audit


Trail (VVPAT) system during the Lok Sabha election 2019.
2. The voter can carry the VVPAT receipt to home.
3. VVPAT slip contains the only name of the candidate a voter
voted for.

Which among the above statements is/are correct?

(a) 1 only
(b) 2 and 3 only
(c) 1 and 3 only
(d) 1, 2 and 3

Ans: (a) 1 only

Learning Zone: When a voter presses a button in the EVM, a


paper slip is printed through the VVPAT.
The slip contains the poll symbol and name of the candidate.
36

It allows the voter to verify his/her choice. After being visible to


Page

the voter from a glass case in the VVPAT for seven seconds, the
ballot slip will be cut and dropped into the drop box in the VVPAT
machine and a beep will be heard.
VVPAT machines can be accessed by polling officers only.
VVPAT is a machine which dispenses a slip with the candidate’s
serial number, name of the candidate and symbol of the party for
which a person has voted for. The slip dropped in a box but the
voter cannot take it home.

Why this question is important? The Election Commission


recently informed the Madras High Court that it had made it
clear way back in 2017 that there shall be 100% use of the Voter
Verifiable Paper Audit Trail (VVPAT) system during the Lok Sabha
election this year to gain voter confidence.

31. With regard to the Small Grants Program (SGP), consider


the following statements

1. It is an initiative of the Global Environment Facility(GEF).


2. It provides financial and technical support to communities
and Civil Society Organizations.
3. It is currently implemented by UNDP.

Which among the above statements is/are correct?

(a) 1 and 2 only


(b) 2 and 3 only
(c) 1 and 3 only
(d) 1, 2 and 3

Ans: (d) 1, 2 and 3


37

Learning Zone: Global Environment Facility GEF Small Grants


Page

Programme (SGP) provides financial and technical support to


communities and Civil Society Organizations to meet the overall
objective of global environmental benefits secured through
community-based initiatives and actions.
It was launched in 1992 with 33 participating countries.
The Program is specifically designed to mobilize bottom-up
actions by empowering local civil society organizations, and poor
and vulnerable communities, including women and Indigenous
Peoples.
It is currently implemented by UNDP on behalf of the GEF
partnership.
The Programme funds grant up to a maximum of $50,000. In
practice, the average grant has been around $25,000. In
addition, the SGP provides a maximum of $150,000 for strategic
projects. These larger projects allow for scaling up and cover a
large number of communities within a critical landscape or
seascape.

Why this question is important? A workshop on Small Grants


Programme (SGP) was recently held in New Delhi.

32. With regard to Ujjwala Utsav, recently seen in the news,


consider the following statements

1. It is to celebrate the stellar role played by various


stakeholders in making PMUY a success.
2. Pradhan Mantri Ujjwala Yojana aims to provide LPG
(liquefied petroleum gas) connections to poor households.
3. The scheme is being implemented by the Ministry of Power.

Which among the above statements is/are correct?


38

(a) 1 and 2 only


Page

(b) 2 and 3 only


(c) 1 and 3 only
(d) 1, 2 and 3

Ans: (a) 1 and 2 only

Learning Zone: Ujjwala Utsav observed recently to celebrate the


stellar role played by various stakeholders in making PMUY a
success.

The event was organised under the aegis of Ministry of Petroleum


&Natural Gas to encourage, motivate as well as felicitate all
frontline field force for their outstanding contribution to PMUY.
The occasion also saw the launch of the PMUY anthem – Ujjwala
Bharat Ujjwala – composed and developed by eminent singer and
film industry personality Padma Shri Kailash Kher.

Why this question is important? Pradhan Mantri Ujjwala


Yojana aims to provide LPG (liquefied petroleum gas) connections
to poor households.
Under the scheme, an adult woman member of a below poverty
line family identified through the Socio-Economic Caste Census
(SECC) is given a deposit-free LPG connection with the financial
assistance of Rs 1,600 per connection by the Centre.

33. With regard to the River Information System, consider


the following statements

1. It is a combination of modern tracking equipment related


hardware and software designed to optimize traffic and
transport.
2. It is implemented by Inland Waterway Authority of India.
39
Page

Which among the above statements is/are correct?


(a) 1 only
(b) 2 only
(c) Both 1 and 2
(d) Neither 1 nor 2

Ans: (c) Both 1 and 2

Learning Zone: It is a combination of modern tracking


equipment related hardware and software designed to optimize
traffic and transport processes in inland navigation.

RIS is being implemented under the overall responsibility of


Inland Waterway Authority of India, a statutory body
administered by the Ministry of Shipping.

Why this question is important? To boost cargo movement on


Ganga, the second phase of river information system (RIS) was
recently inaugurated between Farakka and Patna.

34. With regard to The Global Fund, consider the following


statements

1. It is to Fight AIDS, Tuberculosis and Malaria.


2. It is an initiative under the United Nations.
3. Its headquarters is located at Geneva, Switzerland.

Which among the above statements is/are correct?

(a) 1 and 2 only


(b) 2 and 3 only
40

(c) 1 and 3 only


Page

(d) 1, 2 and 3
Ans: (c) 1 and 3 only

Learning Zone: The Global Fund to Fight AIDS, Tuberculosis


41

and Malaria (or simply the Global Fund) is an international


financing organization that aims to “attract, leverage and invest
Page
additional resources to end the epidemics of HIV/AIDS,
tuberculosis and malaria to support attainment of the
Sustainable Development Goals established by the United
Nations.”

Founded in 2002, the Global Fund is a partnership between


governments, civil society, the private sector and people affected
by the diseases.

The organization maintains its secretariat in Geneva,


Switzerland.

Why this question is important? The Global Fund was formed


as an independent, non-profit foundation under Swiss law and
hosted by the World Health Organization in January 2002. In
January 2009, the organization became an administratively
autonomous organization, terminating its administrative services
agreement with the World Health Organization.

35. With regard to Unlawful Activities (Prevention) Act


(UAPA), consider the following statements

1. Its objective is to make powers available for dealing with


activities directed against the integrity and sovereignty of
India.
2. Under UAPA it is a crime to support any secessionist
movement or to support claims by a foreign power to what
India claims as its territory.

Which among the above statements is/are correct?


42

(a) 1 only
Page

(b) 2 only
(c) Both 1 and 2
(d) Neither 1 nor 2

Ans: (c) Both 1 and 2

Learning Zone: This law is aimed at effective prevention of


unlawful activities associations in India.
Its main objective is to make powers available for dealing with
activities directed against the integrity and sovereignty of India.
The Act makes it a crime to support any secessionist movement
or to support claims by a foreign power to what India claims as
its territory.
The UAPA, framed in 1967, has been amended twice since: first
in 2008 and then in 2012.

Why this question is important? The Union Home Ministry has


banned the Tehreekul-Mujahideen (TuM) under the Unlawful
Activities (Prevention) Act for promoting terrorism and
radicalising and recruiting youth for terrorist activities in India.
Set up in the 1990s, TuM claims to be fighting for the “liberation
of Kashmir.”

36. ‘CULTLASS EXPRESS’ recently seen in news related to:

(a) a multinational training exercise


(b) Transnational train between India and Bangladesh
(c) Army exercise between India and Israel
43

(d) Longest road highway in India


Page
Ans: (a) a multinational training exercise

Learning Zone: It is a multinational training exercise. It is an


exercise designed to assess and improve combined maritime law
enforcement capacity, promote national and regional security in
East Africa as well as information sharing, planning and
operating.

Sponsored by U.S. Africa Command (AFRICOM) and is conducted


by U.S. Naval Forces Africa.

Why this question is important? INS Trikand, a front-line


warship of the Indian Navy, participated in ‘CUTLASS EXPRESS
– 19’ held from 27 Jan to 06 Feb 19.

37. With regard to Solid Fuel Ducted Ramjet (SFDR), consider


the following statements

1. It is a missile propulsion technology jointly developed by


India and Russia.
2. It will enhance their strike range of missile and make them
more lethal.
3. It uses a rotating compressor in its operation.

Which among the above statements is/are correct?

(a) 1 and 2 only


(b) 2 and 3 only
(c) 1 and 3 only
(d) 1, 2 and 3
44
Page

Ans: (a) 1 and 2 only


Learning Zone: Ramjet is a form of air-breathing jet engine that
uses the vehicle’s forward motion to compress incoming air for
combustion without a rotating compressor. Fuel is injected in the
combustion chamber where it mixes with the hot compressed air
and ignites. A ramjet-powered vehicle requires an assisted take-
off like a rocket assist to accelerate it to a speed where it begins
to produce thrust.
Solid Fuel Ducted Ramjet (SFDR): It is a missile propulsion
technology jointly developed by India and Russia.
It is a missile propulsion technology jointly developed by India
and Russia
It will help both India’s surface-to-air and air-to-air missiles to
perform better and enhance their strike range, making them
more lethal.
With it, India can have the fastest long-range missiles in two
categories, providing full-fledged and multi-layered aerial
protection from hostile attacks.

Why this question is important? Defence Research and


Development Organisation (DRDO) successfully flight tested the
second indigenously developed ‘Solid Fuel Ducted Ramjet (SFDR)’
propulsion based missile system.

38. With respect to Convention on the conservation of


migratory species of wild animals(CMS), consider the
following statements

1. It is also referred to as the Bonn Convention.


2. It is the only global and UN-based intergovernmental
organization established exclusively for conservation and
45

management of migratory species.


Page
3. It includes terrestrial, aquatic and avian migratory species
throughout their range.

Which among the above statements is/are correct?

(a) 1 and 2 only


(b) 2 and 3 only
(c) 1 and 3 only
(d) 1, 2 and 3

Ans: (d) 1, 2 and 3

Learning Zone: In order to protect the migratory species


throughout their range countries, a Convention on Conservation
of Migratory Species (CMS), has been in force, under the aegis of
United Nations Environment Programme.
Also referred to as the Bonn Convention, it provides a global
platform for the conservation and sustainable use of migratory
animals and their habitats
CMS is only global and UN-based intergovernmental organization
established exclusively for conservation and management of
terrestrial, aquatic and avian migratory species throughout their
range.

Why this question is important? The 13th Conference of


Parties (COP) of the Convention on the conservation of migratory
species of wild animals (CMS) is going to be hosted by India
during 15th to 22nd February 2020 at Gandhinagar in Gujarat.

India has been a Party to the CMS since 1983. The Conference of
Parties (COP) is the decision-making organ of this convention.
46
Page
39. With regard to National Deworming Day, consider the
following statements

1. It is conducted by the Ministry of Women and Child


Development.
2. It is a single fixed-day approach to treating intestinal worm
infections in all children aged 1- 19 years.
3. Deworming is done through a safe medicine Albendazole.

Which among the above statements is/are correct?

(a) 1 and 2 only


(b) 2 and 3 only
(c) 1 and 3 only
(d) 1, 2 and 3

Ans: (b) 2 and 3 only

Learning Zone: The National Deworming Day is a single fixed-


day approach to treating intestinal worm infections in all children
aged 1- 19 years.
It will mobilize health personnel, state governments and other
stakeholders to prioritize investment in control of Soil-
Transmitted Helminth (STH) infections one of the most common
infections.
All the children are provided deworming tablet in schools and
anganwadis. Besides the deworming tablet, various health
promotion activities related to Water, Sanitation and Hygiene
(WASH) are organised in schools and anganwadis.
The NDD program is a cost-effective program at a scale that
continues to reach crores of children and adolescents with
47

deworming benefits through a safe medicine Albendazole.


Page
Why this question is important? The Ministry of Health and
Family Welfare (MoHFW) is conducting its eighth round of
National Deworming Day (NDD).
NDD is observed bi-annually on 10th February and 10th August
in all states and UTs followed by mop-up activities. This year the
NDD is being conducted on 8th February and mop up day on the
14th February.

40. With regard to the Drugs Technical Advisory Board


(DTAB), consider the following statements

1. It is a statutory body.
2. It works under the Ministry of Health and Family Welfare.

Which among the above statements is/are correct?

(a) 1 only
(b) 2 only
(c) Both 1 and 2
(d) Neither 1 nor 2

Ans: (c) Both 1 and 2

Learning Zone: DTAB is the highest statutory decision-making


body on technical matters related to drugs in the country. It is
constituted as per the Drugs and Cosmetics Act, 1940. It is part
of the Central Drugs Standard Control Organization (CDSCO) in
the Ministry of Health and Family Welfare.

Why this question is important? The Centre, in a notification,


48

has said that medical devices — all implantable devices, CT Scan,


Page

PET and MRI equipment, defibrillators, dialysis machines and


bone marrow separators — will be treated as drugs for human
beings with effect from April 1, 2020.
The decision was taken in consultation with the Drugs Technical
Advisory Board.

41. With regard to Autonomous District Councils in the Sixth


Schedule areas, consider the following statements

1. The governor is empowered to organise and re-organise the


autonomous districts.
2. The acts of Parliament or the state legislature do not apply
to autonomous districts and autonomous regions.
3. The government has introduced a Constitution Amendment
Bill in Rajya Sabha to curtail the financial and executive
powers of the 10 Autonomous Councils.

Which among the above statements is/are correct?

(a) 1 only
(b) 1 and 2 only
(c) 2 and 3 only
(d) 1, 2 and 3

Ans: (b) 1 and 2 only

Learning Zone: The governor is empowered to organise and re-


organise the autonomous districts. Thus, he can increase or
decrease their areas or change their names or define their
boundaries and so on.
49

If there are different tribes in an autonomous district, the


Page

governor can divide the district into several autonomous regions.


The acts of Parliament or the state legislature do not apply to
autonomous districts and autonomous regions or apply with
specified modifications and exceptions.

Why this question is important? The government has


introduced Constitution (125th Amendment) Bill in Rajya Sabha
to increase the financial and executive powers of the 10
Autonomous Councils in the Sixth Schedule areas of the
northeastern region. The amendment will impact one crore tribal
people in Assam, Meghalaya, Tripura and Mizoram.

42. With regard to the Generalised System of Preferences


(GSP) often seen in the news, consider the following
statements:

1. It is to promote economic growth in the developing world by


providing preferential duty-free entry.
2. It is an initiative by WTO.
3. It applies to all the products from the beneficiary countries.

Which among the above statements is/are correct?

(a) 1 and 2 only


(b) 2 and 3 only
(c) 1 only
(d) 1 and 3 only

Ans: (c) 1 only


50

Learning Zone: The Generalized System of Preferences (GSP) is a


Page

U.S. trade program designed to promote economic growth in the


developing world by providing preferential duty-free entry for up
to 4,800 products from 129 designated beneficiary countries and
territories.

Why this question is important? Vowing to reduce the U.S.


deficit with large economies, President Trump is planning to
withdraw the Generalised System of Preferences (GSP) from
India, the world’s largest beneficiary of a scheme that has been in
force since the 1970s.

43. International IP Index 2019 is released by which among


the following?

(a) Global Innovation Policy Center


(b) World Trade Organization
(c) WIPO
(d) Economist Intelligence Unit

Ans: (a) Global Innovation Policy Center

Learning Zone: The International IP Index 2019 is released by


the U.S. Chamber of Commerce’s Global Innovation Policy Center
(GIPC).

Why this question is important? India is placed at rank 36th


this year. In 2018, India was ranked 44 out of 50 countries. In
the first edition of the report in 2014, India had ranked last in
the ranking of 25 countries.
The 2019 Index demonstrates the close correlation between
effective IP protection and economic growth, global
competitiveness, and the creation of 21st-century knowledge-
51

based economies.
Page
44. With regard to the Asiatic Lion Conservation Project,
consider the following statements

1. It is announced by the central government and Gujarat


government.
2. Asiatic Lions are listed as ‘Endangered’ under the IUCN Red
List.

Which among the above statements is/are correct?

(a) 1 only
(b) 2 only
(c) Both 1 and 2
(d) Neither 1 nor 2

Ans: (C) Both 1 and 2

Learning Zone: Asiatic Lions are listed as ‘Endangered’ under


the IUCN Red List.
Its population is restricted to the state of Gujarat in India.
With serious conservation efforts of the State and the Union
Government, the population of Asiatic lions have increased to
over 500 which used to be around 50 by late 1890s.

Why this question is important? The Centre and the Gujarat


government have announced Rs. 97.85 crore Asiatic Lion
Conservation Project.
52

Key aspects of the conservation project include undertaking


Page

“habitat improvement” measures, making more sources of water


available, creating a wildlife crime cell, and a task force for the
Greater Gir region. ‘Greater Gir’ that includes, other than the
existing Gir National Park, sanctuaries in Girnar, Pania and
Mitiyala.
It would also involve having in place a GPS-based tracking
system, which would look at surveillance tracking, animal and
vehicle tracking. There would also be an automated sensor grid
that would have magnetic sensors, movement sensors and
infrared heat sensors.

45. With regard to ‘Nirbhaya Fund’, consider the following


statements

1. It is Rs.1000 crore fund announced in Union Budget 2013.


2. It is administered by the Department of Economic Affairs of
the finance ministry.
3. It is to be utilised for upholding the safety and dignity of
women.

Which among the above statements is/are correct?

(a) 1 and 2 only


(b) 2 and 3 only
(c) 1 and 3 only
(d) 1, 2 and 3

Ans: (d) 1, 2 and 3

Learning Zone: The Rs 1,000 crore Nirbhaya Fund was


announced in Union Budget 2013 by the then Finance Minister P
53

Chidambaram.
Page
The corpus was to be utilised for upholding the safety and dignity
of women.
Ministry of Women and Child Development apart from several
other concerned ministries were authorised to work out details of
structure, scope and application of this fund.
The Fund is administered by the Department of Economic Affairs
of the finance ministry.

Why this question is important? A parliamentary panel has


taken strong exception to the utilisation of the Nirbhaya Fund in
the construction of buildings, saying such allocations defeat the
very purpose of the project — that of safety for women.

46. Which among the following statements is/are correct


regarding India-Norway Marine Pollution Initiative?

1. It is an initiative launched by The Union Ministry of


Environment, Forests and Climate Change and Norwegian
Ministry of Foreign Affairs.
2. It works towards beach clean-up efforts, awareness-raising
campaigns and using plastic waste as fuel substitution for
coal in cement production.
3. This initiative will seek to support local governments in
implementing sustainable waste management practices.

Select the correct answer using the codes given below:

(a) 1 and 2 only


(b) 2 and 3 only
(c) 1 and 3 only
(d) 1, 2 and 3
54
Page

Ans: (d) 1, 2 and 3


Learning Zone: The Initiative will combat marine pollution,
which is one of the fastest growing environmental concerns.
Through a range of implementing partners, this initiative will
seek to support local governments in implementing sustainable
waste management practices, develop systems for collecting and
analysing information about sources and scope of marine
pollution.
They will also work towards beach clean-up efforts, awareness
raising campaigns and pilot project using plastic waste as fuel
substitution for coal in cement production.

Why this question is important? The Union Ministry of


Environment, Forests and Climate Change has signed a Letter of
Intent (LoI) with the Norwegian Ministry of Foreign Affairs to
launch the ‘India-Norway Marine Pollution Initiative’.

47. With regard to ‘GiveItUp’ campaign, consider the


following statements

1. It encourages well-to-do households to mandatorily give up


their liquefied petroleum gas (LPG) subsidy.
2. The money surrendered under this movement will be
utilized for the poor to get LPG connection in rural areas
only.

Which among the above statements is/are correct?

(a) 1 only
(b) 2 only
55

(c) Both 1 and 2


Page

(d) Neither 1 nor 2


Ans: (d) Neither 1 nor 2

Learning Zone: ‘Give it Up’ scheme encourages well-


well-to-do
households to voluntarily give up their liquefied petroleum gas

(LPG) subsidy so that it could be targeted to the poor who remain


56

reliant on polluting cooking fuels such as wood, dung, crop


residues and coal.
Page
The money surrendered under this movement will be utilised for
poor to get LPG connection in rural as well as in urban areas who
are still using firewood for cooking.

Why this question is important? As on 06.02.2019, nearly 1.04


crore LPG consumers have voluntarily surrendered their LPG
subsidy under ‘GiveItUp’ campaign.

48. With regard to Innovations For Defence Excellence


(iDEX), consider the following statements

1. Its aims at the creation of an ecosystem to foster innovation


and technology development in Defence and Aerospace by
engaging Industries including MSMEs, start-ups, individual
innovators etc
2. It provides them with grants/funding and other support to
carry out R&D.
3. It will be managed and funded by the Ministry of Home
Affairs.

Which among the above statements is/are correct?

(a) 1 and 2 only


(b) 2 and 3 only
(c) 1 and 3 only
(d) 1, 2 and 3

Ans: (a) 1 and 2 only


57
Page
Learning Zone: Innovations for Defence Excellence (iDEX)
launched by the Government in April 2018, primarily aims at
creation of an ecosystem to foster innovation and technology
development in Defence and Aerospace by engaging Industries
including MSMEs, start-ups, individual innovators, R&D
institutes & academia, and provide them grants/funding and
other support to carry out R&D which has good potential for
future adoption for Indian defence and aerospace needs.

iDEX will be funded and managed by a ‘Defence Innovation


Organization (DIO)’ which has been formed as a ‘not for profit’
company as per Section 8 of the Companies Act 2013.

Why this question is important? The Defence Innovation


Organisation set up under Innovations for Defence Excellence
(iDEX) framework has announced setting up of two Defence
Innovation Hubs (DIHs) in Tamil Nadu (Coimbatore) and
Maharashtra (Nashik).

49. With regard to Circular Economy, consider the following


statements

1. It follows the principle of preservation and enhancement of


natural capital by controlling finite stocks.
2. It has the potential to increase productivity and create jobs.
3. It reduces carbon emissions and preserve valuable raw
materials.
58

Which among the above statements is/are correct?


Page
(a) 1 and 2 only
(b) 2 and 3 only
(c) 1 and 3 only
(d) 1, 2 and 3

Ans: (d) 1, 2 and 3

Learning Zone: A circular economy is an alternative to a


traditional linear economy (make, use, dispose of) in which we
keep resources in use for as long as possible, extract the
maximum value from them whilst in use, then recover and
regenerate products and materials at the end of each service life.
It follows the principle of preservation and enhancement of
natural capital by controlling finite stocks and balancing
renewable resource flows. The other principles suggest optimizing
of resource yields by circulating products, components, and
materials at their highest utility at all times, in both technical
and biological cycles.

The circular economy has the potential to increase productivity


and create jobs, whilst reducing carbon emissions and preserving
valuable raw materials.

Why this question is important? National Productivity Council


(NPC), an autonomous registered society under Department for
Promotion of Industry and Internal Trade, Ministry of Commerce
& Industry, NPC is celebrating its 61st Foundation Day on 12th
February with the theme “Circular Economy for Productivity &
Sustainability”.
59
Page
50. With regard to the LAWASIA Human Rights Conference
recently held, consider the following statements

1. It was the first LAWASIA Human Rights Conference held in


India.
2. LAWAISA is a non-governmental organization of human
rights activists.

Which among the above statements is/are correct?

(a) 1 only
(b) 2 only
(c) Both 1 and 2
(d) Neither 1 nor 2

Ans: (a) 1 only

Learning Zone: LAWASIA is a regional association of lawyers,


judges, jurists and legal organisations and it advocates for the
interests and concerns of the Asia Pacific legal profession.
It provides a platform to promote the cross-jurisdictional
exchange of legal knowledge; as a voice of the legal profession;
and as a conduit for encouraging adherence to mutually-held
principles of the rule of law, professional integrity and the
protection of human rights.

Why this question is important? The first LAWASIA Human


Rights Conference was organised by LAWASIA, in association
with the Bar Association of India recently.
60
Page
The conference aims to provide a unique opportunity for lawyers
and associated professional members to exchange insights and
expertise on topics of significant importance to all.

51. With regard to the World Gold Council, consider the


following statements?

1. It works across all parts of the industry, from gold mining to


investment.
2. Its aim is to stimulate and sustain demand for gold.
3. Its headquarters is located at Geneva.

Which among the above statements is/are correct?

(a) 1 and 2 only


(b) 2 and 3 only
(c) 1 and 3 only
(d) 1, 2 and 3

Ans: (a) 1 and 2 only

Learning Zone: The World Gold Council is the market


development organisation for the gold industry. It works across
all parts of the industry, from gold mining to investment, and
their aim is to stimulate and sustain demand for gold.

They frequently publish research that demonstrates gold’s


strength as a preserver of wealth – both for investors and
countries. They also provide analysis of the industry, offering
61

insights into the drivers of gold demand. They have also launched
Page
various products such as SPDR GLD and gold accumulation
plans in India and China.

Headquartered in the London United Kingdom, they have offices

62

in India, China, Singapore, Japan and the United States


Page
Why this question is important?

WGC has released a report on gold demand in 2019.

52. With regard to recently launched National Clean Air


Programme (NCAP), consider the following statements

1. It is under the Environment Protection Act 1986.


2. It aims to achieve a national-level target of 20-30%
reduction of PM2.5 and PM10 concentration by between
2017 and 2024.

Which among the above statements is/are correct?

(a) 1 only
(b) 2 only
(c) Both 1 and 2
(d) Neither 1 nor 2

Ans: (b) 2 only

Learning Zone:

Key features of the National Clean Air Program (NCAP):

● Achieve a national-level target of 20-30% reduction of PM2.5


63

and PM10 concentration by between 2017 and 2024.


Page
● Central Pollution Control Board (CPCB) will execute this
nation-wide programme in consonance with section 162 (b)
of the Air (Prevention and Control of Pollution) Act, 1986.
● The programme has been launched with an initial budget of
₹300 crore for the first two years.
● The plan includes 102 non-attainment cities, across 23
states and Union territories, which were identified by the
Central Pollution Control Board (CPCB) on the basis of their
ambient air quality data between 2011 and 2015.
● Non-attainment cities are those which have been
consistently showing poorer air quality than the National
Ambient Air Quality Standards. These include Delhi,
Varanasi, Bhopal, Kolkata, Noida, Muzaffarpur, and
Mumbai.
● As part of the programme, the Centre also plans to scale up
the air quality monitoring network across India.

Why this question is important?

The government has announced the National Clean Air Program


(NCAP). This is the first ever effort in the country to frame a
national framework for air quality management with a time-
bound reduction target.

The program will not be notified under the Environment


Protection Act or any other Act to create a firm mandate with a
64

strong legal back up for cities and regions to implement NCAP in


Page

a time bound manner for effective reduction.


53. With regard to ECONiwasSamhita 2018, consider the
following statements

1. It is the Energy Conservation Building Code for commercial


buildings.
2. It is launched by the Ministry of Power.

Which among the above statements is/are correct?

(a) 1 only
(b) 2 only
(c) Both 1 and 2
(d) Neither 1 nor 2

Ans: (b) 2 only

Learning Zone: ECO NiwasSamhita 2018 an Energy


Conservation Building Code for residential buildings, to push for
energy efficiency in the residential sector was launched on
December 14, 2018. It aims to promote design and construction
of homes including apartments and townships to give benefits of
energy efficiency to the occupants. Ministry of Power launched
the ECO NiwasSamhita 2018.

Why this question is important?

Bureau of Energy Efficiency and CPWD sign MoU on promoting


energy efficiency in buildings
65

According to the MoU, BEE and CPWD will cooperate on


Page

promoting designs and construction of Energy Conservation


Building Code (ECBC) compliant new buildings, star rating of
CPWD managed buildings across the country with no registration
or renewal fee, awareness on energy efficiency in building sector
and support for capacity building of CPWD officials in ECBC.

54. Which among the following event would happen to the


earth if its poles are flipped?

1. Higher amounts of solar radiation at the surface


2. Change in Climate
3. Strong winds
4. Electric grid collapse

Select the correct answer using the codes given below:

(a) 1 and 2 only


(b) 3 and 4 only
(c) 1,2 and 4 only
(d) 2,3 and 4 only

Ans: (c) 1,2 and 4 only

Learning Zone: Scientists in recent years have predicted that


Earth’s magnetic field could be gearing up to ‘flip’ – a shift in
which the magnetic south pole would become magnetic north,
and vice versa. Such an event could have catastrophic effects,
wreaking havoc on the electric grid and leaving life at the surface
exposed to higher amounts of solar radiation.
66

Electric grid collapse from severe solar storms is a major risk. As


the magnetic field continues to weaken, scientists are
Page
highlighting the importance of off-the-grid energy systems using
renewable energy sources to protect the Earth against a blackout.

Very highly charged particles can have a deleterious effect on the


satellites and astronauts.

Earth’s climate could also change. A recent Danish study has


found that the earth’s weather has been significantly affected by
the planet’s magnetic field.

Why this question is important?

Researchers say Earth’s magnetic North Pole is ‘skittering’ away


from Canada, towards Siberia.

55. Section 126 of the Representation of People Act, recently


seen in the news, refers to:

(a) Prohibits conduct of Exit poll and dissemination of their


results
(b) Model code of conduct
(c) Election expenses
(d) Scrutinizing the election nomination of a candidate

Ans: (a) Prohibits conduct of Exit poll and dissemination of


their results

Learning Zone: Section 126 of the RP Act prohibits displaying


67

any election matter by means, inter alia, of television or similar


Page
apparatus, during the period of 48 hours before the hour fixed for
conclusion of poll in a constituency.

“Election matter” has been defined in that Section as any


matter intended or calculated to influence or affect the result of
an election.

The provision prohibits the conduct of Exit poll and


dissemination of their results during the period mentioned
therein, in the hour fixed for the commencement of polls in the
first phase and half hour after the time fixed for the close of the
poll for the last phase in all the States.

Violation of the provisions of Section 126 is punishable with


imprisonment up to a period of two years, or with fine or both.

Why this question is important?

A Committee constituted to review and suggest modifications and


changes in the provisions of Section 126 and other sections of the
RP Act 1951 and provisions of Model Code of Conduct have
submitted its report. The panel was headed by senior deputy
election commissioner Umesh Sinha.

56. With regard to Sino-Indian Digital Collaboration Plaza


(SIDCOP), consider the following statements

1. It is an initiative of NASSCOM alone.


2. It facilitates Chinese IT enterprises to do business in India
68

freely.
Page
Which among the above statements is/are correct?

(a) 1 only
(b) 2 only
(c) Both 1 and 2
(d) Neither 1 nor 2

Ans: (d) Neither 1 nor 2

Learning Zone: The initiative aims to bring Indian IT companies


and Chinese enterprises closer to each other on a single AI
enabled platform. This platform will be managed by a joint
venture comprising of one Indian and Chinese company.

The initiative aims to encash the expertise of Indian IT


enterprises in business transformation and operational
optimization by using IT tools in complex business environments.

It offers a boundary-less marketplace for Chinese enterprises in


order to assist them in operational optimization and adopting
industry best practices in business solutions by connecting with
Indian enterprises.

Why this question is important?

The Sino-Indian Digital Collaboration Plaza (SIDCOP) has been


launched by the National Association of Software and Services
Companies (NASSCOM) in association with Municipal
Governments of Guiyang and Dalian.
69
Page
57. Partnership Summit, often seen in news is organized by
which among the following?

1. NITI Aayog
2. DIPP
3. CII
4. World Bank

Select the correct answer using the codes given below:

(a) 1, 3 and 4 only


(b) 2 and 3 only
(c) 1, 2 and 3 only
(d) 1, 2, 3 and 4

Ans: (b) 2 and 3 only

Learning Zone: It is being organized by the Department of


Industrial Policy and Promotion, Ministry of Commerce &
Industry, Government of India, State Government of Maharashtra
and Confederation of Indian Industry.

The Partnership Summit provides for a global platform to


dialogue, debate, deliberate and engage Indian and global leaders
on economic policy and growth trends in India.

Why this question is important?

The 25th edition of the Partnership Summit will be held in


70

Mumbai recently.
Page
58.‘Women of India Organic Festival’ is organized by which
among the following?

(a) Ministry of Women and Child Development


(b) Ministry of social justice and empowerment
(c) Ministry of Agriculture & Farmers' Welfare
(d) Ministry of Food Processing Industries

Ans: (a) Ministry of Women and Child Development

Learning Zone: The Ministry of Women and Child Development


is organising the three-day 6th ‘Women of India Organic Festival’
in Chandigarh, to celebrate and promote women farmers and
entrepreneurs in the organic sector from the remotest parts of
India.

Why this question is important?

This initiative is a move to benefit women entrepreneurs, and


also actively promote organic food and products.

The Women of India Festival is also intended to highlight the


health and environmental advantages of organic goods, provide a
platform for women engaged in this economy and encourage the
development of sustainable and easily accessible sales outlets for
organic producers from remote areas.
71

59. With regard to PradhanMantri MUDRA Yojana (PMMY)


Page

scheme, consider the following statements


1. The scheme’s objective is to refinance collateral-free loans
given by the lenders to small borrowers.
2. Mudra Loans are available for non-agricultural activities
uptoRs. 10 lakh.

Which among the above statements is/are correct?

(a) 1 only
(b) 2 only
(c) Both 1 and 2
(d) Neither 1 nor 2

Ans: (c) Both 1 and 2

Learning Zone:

The PMMY Scheme was launched in April 2015. The scheme’s


objective is to refinance collateral-free loans given by the lenders
to small borrowers.

The scheme, which has a corpus of Rs 20,000 crore, can lend


between Rs 50,000 and Rs 10 lakh to small entrepreneurs.

Banks and MFIs can draw refinance under the MUDRA Scheme
after becoming member-lending institutions of MUDRA.

Mudra Loans are available for non-agricultural activities uptoRs.


10 lakh and activities allied to agriculture such as Dairy, Poultry,
72

Bee Keeping etc, are also covered.


Page
Mudra’s unique features include a Mudra Card which permits
access to Working Capital through ATMs and Card Machines.

Why this question is important?

The finance ministry has asked the banks to review all loans

73

sanctioned under the PradhanMantri Mudra Yojana (PMMY or


Page
Mudra loan scheme), as the non-performing assets (NPA) have
crossed Rs 11,000 crore within three years of the launch of the
scheme.

60. One family one job scheme, recently seen in the news, is
a scheme of which state?

(a) Kerala
(b) Uttar Pradesh
(c) Sikkim
(d) Telangana

Ans: (c) Sikkim

Learning Zone: Sikkim has announced the One family one job
scheme to provide employment to the youth of Sikkim.

One family one job scheme entitles one government job for every
family in the state.

Eligibility: Only members of those families which do not have a


government job at present are eligible for government employees
under the scheme.

The government has announced that the nature of the job is not
temporary and would be regularised after five years.
74

Why this question is important?


Page
Sikkim is the first state in the country to launch an exclusive
program which entitles every family in the state with a
government job and Sikkim earmarks 70% of its revenues
towards salaries for state government employees.

61. With regard to the National Wildlife Board, consider the


following statements

1. It is a statutory organization constituted under the Wildlife


Protection Act, 1972.
2. It is chaired by the Prime Minister.
3. It approves projects in and around national parks and
sanctuaries.

Which among the above statements is/are correct?

(a) 1 and 2 only


(b) 2 and 3 only
(c) 1 and 3 only
(d) 1, 2 and 3

Ans: (d) 1,2 and 3

Learning Zone: National Board for Wild Life is a statutory


organization constituted under the Wildlife Protection Act, 1972.

It is an apex body to review all wildlife-related matters and


approves projects in and around national parks and sanctuaries.
75
Page
Composition: The National Board for Wild Life is chaired by the
Prime Minister, Union Minister of Environment, Forest and
Climate Change is the vice-chairman of the Board and the
members include 15 non-government members, 19 ex-officio
members and 10 government officials such as secretaries.

Why this question is important?

The National Wildlife Board has given its approval for the Trishna
Gas project of ONGC which falls in the Trishna Wildlife
Sanctuary in the Gomati district of Tripura.

ONGC has discovered 10-12 gas bearing wells in the Trishna


Wildlife sanctuary.

62. With regard to the Asian Waterbird Census (AWC),


consider the following statements

1. It happens annually.
2. In India, it is coordinated by the Bombay Natural history
Society (BNHS) and BirdLife International.
3. Its main focus is to monitor the status of all the birds and
the wetlands.

Which among the above statements is/are correct?

(a) 1 only
(b) 2 and 3 only
(c) 1 and 3 only
(d) 1, 2 and 3
76
Page

Ans: (a) 1 only


Learning Zone: Asian Waterbird Census is an annual event in
which thousands of volunteers across Asia and Australasia count
waterbirds in the wetlands of their country. This event happens
every January. This event is coordinated by Wetlands
International and forms part of the global waterbird monitoring
program called the International Waterbird Census (IWC).

Asian Waterbird Census (AWC) was started in the year 1987. Its
main focus is to monitor the status of waterbirds and the
wetlands.
In India, the AWC is annually coordinated by the Bombay
Natural history Society (BNHS) and Wetlands International.

Why this question is important?

A waterbird survey conducted in the Upper Kuttanad region of


Kerala has recorded 16,767 birds of 47 continental and local
species.

63. Consider the following pairs

Harvest festival State

1. Pongal - Tamil Nadu


2. MaghaBihu - Punjab
3. Uttarayan - Gujarat
4. Maghi - Himachal Pradesh

Which among the above pairs is/are correctly matched?


77
Page

(a) 1 and 3 only


(b) 2 and 4 only
(c) 1, 3 and 4 only
(d) 1 and 4 only

Ans :( a) 1 and 3 only

Learning Zone:

Various festivals being celebrated across the Nations:

MakarSankranti: The festival of MakarSankranti is being


celebrated today when the Sun enters the Makar zodiac and the
days begin to lengthen compared to nights.

Pongal: In South India and particularly in Tamil Nadu, it’s the


festival of Pongal which is being celebrated over 4 days at harvest
time.

MaghaBihu: In Assam and many parts of the North East, the


festival of MaghaBihu is celebrated. It sees the first harvest of the
season is offered to the gods along with prayers for peace and
prosperity.

Uttarayan: Gujarat celebrates it in the form of the convivial kite


festival of Uttarayan.
78
Page
Maghi: In Punjab, MakarSankranti is celebrated as Maghi.
Bathing in a river in the early hours on Maghi is important.

Saaji: In Shimla District of Himachal Pradesh, MakaraSankranti


is known as MaghaSaaji. Saaji is the Pahari word for Sankranti,
the start of the new month. Hence this day marks the start of the
month of Magha.

Kicheri: The festival is known as Kicheri in Uttar Pradesh and


involves ritual bathing.

64. With regard to District mineral foundations, consider the


following statements

1. It works for the interest of the benefit of the persons and


areas affected mining-related operations.
2. It is a statutory body.

Which among the above statements is/are correct?

(a) 1 only
(b) 2 only
(c) Both 1 and 2
(d) Neither 1 nor 2

Ans: (c) Both 1 and 2

Learning Zone: DMFs were instituted under the Mines and


Minerals (Development and Regulation) (MMDR) Amendment Act
79

2015 as non-profit trusts to work for the interest and benefit of


Page

persons and areas affected by mining-related operations.


The objective of District Mineral Foundation is to work for the
interest of the benefit of the persons and areas affected mining
mining-

related operations in such manner as may be prescribed by the


State Government.
80

Why this question is important?


Page
The Odisha government is planning to move its district mineral
foundations (DMF) to its steel and mines department from the
planning and convergence department. The plan to move the
DMFs aims to improve the implementation and use of funds.

65.With regard to Formalin, often seen news, consider the


following statements

1. It is a toxic colourless solution.


2. It is used to extend the storage of fish.
3. It can cause irritation in the eyes, throat, skin and stomach.

Which among the above statements is/are correct?

(a) 1 and 3 only


(b) 2 only
(c) 2 and 3 only
(d) 1, 2 and 3

Ans: (d) 1, 2 and 3

Learning Zone: Formalin is a toxic, colourless solution that is


derived by dissolving formaldehyde gas in water.

It is a cancer-inducing chemical used to preserve fish and is used


as a disinfectant. It is used in the manufacture of pesticides,
81

fertilisers, glue, paper and paint, among other products.


Page
Formalin causes irritation in the eyes, throat, skin and stomach.
In the long run, continued exposure causes harm to the kidneys,
liver and can even cause cancers.

Formaldehyde is a highly reactive, flammable gas, which means it


can become a fire hazard when exposed to flame or heat.

Why this question is important?

Bihar’s health department has imposed a blanket ban for 15 days


on sale of fish from Andhra Pradesh and West Bengal in capital
Patna after samples were found to be contaminated with
formalin. The ban includes storage and transportation of fish
from Andhra Pradesh and West Bengal.

66. With regard to ‘Raisina Dialogue’, consider the following


statements

1. It is a meeting of Ministers of External Affairs.


2. It is designed to explore prospects and opportunities for
Asian integration.
3. It is constituted in 1950.

Which among the above statements is/are correct?

(a) 1 and 2 only


(b) 2 and 3 only
(c) 1 and 3 only
82

(d) 2 only
Page
Ans: (d) 2 only

Learning Zone:

This is an annual geopolitical event, organised by the Ministry of


External Affairs and Observer Research Foundation (ORF).

It is designed to explore prospects and opportunities for Asian


integration as well as Asia’s integration with the larger world. It is
predicated on India’s vital role in the Indian Ocean Region and
how India along with its partners can build a stable regional and
world order.

Participants: The conference is a multi-stakeholder, cross-


sectoral meeting involving policy and decision-makers, including
but not limited to Foreign, Defence and Finance Ministers of
different countries, high-level government officials and policy
practitioners, leading personalities from business and industry,
and members of the strategic community, media and academia.

Why this question is important?

The fourth edition of the Raisina Dialogue, India’s flagship


annual conference on geopolitics and geo-economics, titled ‘World
Reorder’, was recently held in New Delhi.

67. Which among the following is not a species of a


crocodile?

(a) Mugger
83

(b) Saltwater
Page

(c) Silonia
(d) Gharial

Ans: (c) Silonia

Learning Zone:

There are three species of crocodilians—saltwater, Mugger and


Gharial.

Mugger:

 The mugger crocodile also called the Indian crocodile, or


marsh crocodile, is found throughout the Indian
subcontinent.
 It is listed as vulnerable by IUCN.
 The mugger is mainly a freshwater species and found in
lakes, rivers and marshes.

Gharial:

 The Gharial or fish-eating crocodile is native to the Indian


subcontinent.
 It is listed as a Critically Endangered by IUCN.
 Small released populations are present and increasing in the
rivers of the National Chambal Sanctuary, Katarniaghat
Wildlife Sanctuary, Son River Sanctuary and the rainforest
84

biome of Mahanadi in Satkosia Gorge Sanctuary, Orissa.


Page
Saltwater Crocodile:

 It is the largest of all living reptiles. It listed as least concern by


IUCN. It is found throughout the east coast of India.

Silonia is a genus of schilbid catfishes native to Asia.

Why this question is important?

Recent Crocodile Census by Odisha.

Findings: The population of the saltwater or estuarine crocodile


(Crocodylusporosus) has increased in the water bodies of
Odisha’sBhitarkanika National Park and its nearby areas in
Kendrapara district.

1,742 individuals have been recorded in this year’s annual reptile


census.
The increase in population was primarily due to the far-sighted
measures of the government.

68. Turtle Festival, recently held in Odisha is organized by


which among the following?

1. Ministry of Culture
2. Cox & Kings Foundation
3. UNEP
4. Action for Protection of Wild Animals (APOWA)
85
Page

Select the correct answer using the codes given below:


(a) 1, 2 and 4 only
(b) 2 and 4 only
(c) 1, 2 and 3 only
(d) 3 and 4 only

Ans: (b) 2 and 4 only

Learning Zone: First Odisha Turtle Festival was held recently in


Puri to create awareness and promote ideas on conservation of
olive ridley turtles.

It is organised by Cox & Kings Foundation in association with


Humane Society International/India and Action for Protection of
Wild Animals (APOWA).

Why this question is important?

Odisha has half of the world’s Olive Ridley turtle population and
90% of India’s turtle population lives in the state.

69. Philip Kotler Presidential Award by the World Marketing


Summit India, is given to which among the following?

(a) Ministry of Commerce And Industry


(b) Donald Trump
(c) Narendra Modi
(d) Reliance India
86

Ans: (c) Narendra Modi


Page
Learning Zone: Prime Minister Narendra Modi was awarded first
ever Philip Kotler Presidential Award. He was selected for his
outstanding leadership for the nation.

The Philip Kotler Presidential Award recognises and celebrates


achievements of organisations, marketing teams, and individuals
in different industries around the world.

Philip Kotler is a world-renowned Professor of Marketing at


Northwestern University, Kellogg School of Management

70. With regard to SwadeshDarshan Scheme, consider the


following statements

1. It is a 100% centrally funded scheme.


2. It is a scheme under the Ministry of Tourism.

Which among the above statements is/are correct?

(a) 1 only
(b) 2 only
(c) Both 1 and 2
(d) Neither 1 nor 2

Ans: (c) Both 1 and 2


87
Page
Learning Zone: The scheme is 100% centrally funded for the
project components undertaken for public funding.

It leverages the voluntary funding available for Corporate Social


Responsibility (CSR) initiatives of Central Public Sector
Undertakings and corporate sector.

Funding of individual project will vary from state to state and will
be finalised on the basis of detailed project reports prepared by
PMC (Programme Management Consultant).
A National Steering Committee (NSC) will be constituted with
Minister in charge of M/O Tourism as Chairman, to steer the
mission objectives and vision of the scheme.

A Mission Directorate headed by the Member Secretary, NSC as a


nodal officer will help in identification of projects in consultation
with the States/ UTs governments and other stakeholders.

PMC will be a national level consultant to be appointed by the


Mission Directorate.

Why this question is important?

Prime Minister Narendra Modi recently inaugurated a project for


the development of spiritual circuit comprising three important
pilgrimage centres of Sree Padmanabha Swamy Temple,
Aranmula and Sabarimala in Kerala.

The SreePadmanabaswamy Temple is one of the 108


divyadesams of Lord Vishnu.
The project is being implemented under the SwadeshDarshan
88

scheme
Page
71. Which among the following statements is/are true
regarding Womaniya on GeM initiative recently seen in the
news?

1. Its objective is to achieve gender-inclusive economic growth.


2. It will provide skill development courses to women from
rural areas.
3. It helps women, self-help groups, to sell handicrafts and
handloom, jute and coir products etc.

Select the correct answer using the codes given below:

(a) 1 and 2 only


(b) 2 and 3 only
(c) 1 and 3 only
(d) 1, 2 and 3

Ans: (c) 1 and 3 only

Learning Zone: The initiative – Womaniya on GeM – seeks to


develop women entrepreneurship on the margins of society to
achieve gender-inclusive economic growth.

The initiative would enable women entrepreneurs and women


self-help groups to sell handicrafts and handloom, jute and coir
products, home décor and office furnishings, directly to various
government ministries, departments and institutions.
89

Why this question is important?


Page
Since women tend to invest up to 90% of their earnings back in
their families to provide better nutrition, health care and
education to their children, their economic empowerment is
essential for poverty alleviation which would be made possible
through Womaniya initiative.

Womaniya on GeM will address goals and objectives under


United Nations Sustainable Development Goal 5: Achieve gender
equality and empower all women and girls.

72. With regard to the Global Housing Technology Challenge


(GHTC), consider the following statements

1. It is an initiative under the United Nations Human


Settlement Programme.
2. It aims to fast-track the construction of affordable housing
and meet the target of constructing 1.2 crore houses by
2022.

Which among the above statements is/are correct?

(a) 1 only
(b) 2 only
(c) Both 1 and 2
(d) Neither 1 nor 2

Ans: (b) 2 only


90
Page
Learning Zone: Government has launched the Global Housing
Technology Challenge. The challenge is undertaken under the
PradhanMantriAwasYojana Urban (PMAY-U).

Why this question is important?

GHTC aims to fast-track the construction of affordable housing


and meet the target of constructing 1.2 crore houses by 2022.

GHTC focuses on identifying and mainstreaming proven


demonstrable technologies for lighthouse projects and spotting
potential future technologies for incubation and acceleration
support through ASHA (Affordable Sustainable Housing
Accelerators) — India.

73. With regard to Congenital Central Hypoventilation


Syndrome (CCHS), consider the following statements

1. It is a disorder of the nervous system.


2. It is a commonly occurring disease across the world.

Which among the above statements is/are correct?

(a) 1 only
(b) 2 only
(c) Both 1 and 2
91

(d) Neither 1 nor 2


Page
Ans: (a) 1 only

Learning Zone: CCHS is a disorder of the nervous system in


which the cue to
o breathe is lost when the patient goes to sleep.

This results in a lack of oxygen and a buildup of carbon dioxide


in the body, which can sometimes turn fatal. There are less than
92

1,000 known cases all over the world.


Page
Though the name describes the disorder as congenital, some
forms of the disease may also be present in adults. In fact, adult
onset is far more common than congenital presentation; there
have been many adult cases reported in medical journals over the
years.

The disease is also known as Ondine’s Curse.

Why this question is important?

An infant in Delhi is suffering from this rare disease.

74. With regard to Plastic Pollution, consider the following


statements

1. “Beat Plastic Pollution” was the theme for World


Environment Day 2018.
2. India pledged to eliminate all forms of plastic by 2022.
3. 22 states in India has announced a ban single-use plastics.

Which among the above statements is/are correct?

(a) 1 and 2 only


(b) 2 and 3 only
93

(c) 1, 2 and 3
(d) 1 and 3
Page
Ans: (d) 1 and 3

Learning Zone: India has also won global acclaim for its “Beat
Plastic Pollution” resolve declared on World Environment Day last
year, under which it pledged to eliminate single-use plastic by
2022.

“Beat Plastic Pollution”, the theme for World Environment Day


2018, is a call to action for all of us to come together to combat
one of the great environmental challenges of our time.

Why this question is important?

So far, 22 States and Union Territories have joined the fight to


beat the plastic pollution, announcing a ban on single-use
plastics such as carry bags, cups, plates, cutlery, straws and
thermocol products. Puducherry will implement a ban from
March 1.

75. With regard to City Momentum Index, consider the


following statements

1. It is released by UN-Habitat.
2. Bengaluru is the world’s most dynamic city according to
Index.
94

Which among the above statements is/are correct?


Page
(a) 1 only
(b) 2 only
(c) Both 1 and 2
(d) Neither 1 nor 2

Ans: (b) 2 only

Learning Zone: 6th City Momentum Index has been released by


JLL. It measures momentum for 131 of the world’s most
commercially active cities. This is done by tracking a range of
socio-economic and commercial real estate indicators over a
three-year period to identify the urban economies and real estate
markets undergoing the most rapid expansion.

Why this question is important?

Silicon Valley of India, Bengaluru has emerged as the world’s


most dynamic city.

76. With regard to Directions issued by the Supreme Court in


appointing DGP, consider the following statements

1. The state shall send names of senior police officers to the


Union Public Service Commission (UPSC).
2. The states are free to appoint any one of the senior police
officers as DGP.
95

Which among the above statements is/are correct?


Page
(a) 1 only
(b) 2 only
(c) Both 1 and 2
(d) Neither 1 nor 2

Ans: (a) 1 only

Learning Zone: Directions issued by the Supreme Court in


appointing DGPs:

 States and Union Territories shall send names of senior police


officers to the Union Public Service Commission (UPSC) for
being considered as probable candidates for the post of DGPs
or police commissioners.
 The UPSC would then prepare a list of three most suitable
candidates out of the list of names sent by states and Union
Territories.
 The states are free to appoint any one of them as the police
chief.
 It is mandatory for the states to send the list of senior police
officers to the UPSC at least three months prior to the
retirement of the incumbent. The UPSC would then form a
committee and intimate the state concerned, which in turn will
immediately appoint one of the persons from among that list.
96

Why this question is important?


Page
The Supreme Court has dismissed the pleas of the states of
Punjab, Kerala, West Bengal, Haryana and Bihar which sought
implementation of their local laws regarding the selection and
appointment of DGPs.

77. With regard to the National Action Plan for Drug Demand
Reduction (2018-2023), consider the following statements

1. It is drafted by the Ministry of Social Justice and


Empowerment.
2. It involves education, de-addiction and rehabilitation of
affected individuals and their families to address the issue.
3. It seeks to increase community participation and public
cooperation in the reduction of demand by involving
Panchayati Raj institutions, Urban Local Bodies etc.

Which among the above statements is/are correct?

(a) 1 only
(b) 2 and 3 only
(c) 1, 2 and 3
(d) 1 and 2 only

Ans: (c) 1, 2 and 3

Learning Zone:
The components of the National Action Plan for Drug Demand
97

Reduction (2018-2023) are:


Page
Aim: employ a multi-pronged strategy involving education, de-
addiction and rehabilitation of affected individuals and their
families to address the issue.

Focus on preventive education, awareness generation,


counselling, treatment and rehabilitation of drug-dependent
people, besides training and capacity-building of service
providers through the collaborative efforts of the Centre, state
and NGOs.

Increase community participation and public cooperation in the


reduction of demand by involving Panchayati Raj institutions,
Urban Local Bodies, Nehru Yuva Kendra Sangathan and other
local groups like MahilaMandals, self-help groups etc to tackle
the menace of drugs.

Why this question is important?

The Ministry of Social Justice and Empowerment has drafted


National Action Plan for Drug Demand Reduction (2018-2023) for
addressing the problem of drug and substance abuse in the
country, dumping a long-pending draft policy on the matter.

78. With regard to the Right of Children to Free and


Compulsory Education (Amendment) Act, 2019, consider the
98

following statements
Page
1. It would be left to the states to decide whether to continue
the no-detention policy.
2. If a child fails, the amendment bill grants a provision to give
her or him additional opportunity to take a re-examination
within two months.
3. It is based on the recommendation ofthe TSR Subramanian
committee.

Which among the above statements is/are correct?

(a) 1 and 2 only


(b) 2 and 3 only
(c) 1 and 3 only
(d) 1, 2 and 3

Ans: (d) 1,2 and 3

Learning Zone:

RTE amendment Bill- Key features:

● The Bill seeks to amend the Right to Education (RTE) Act to


abolish the “no-detention” policy in schools. Under the
current provisions of the Act, no student can be detained up
99

to class VIII.
Page
● As per the amendment, it would be left to the states to
decide whether to continue the no-detention policy.
● The bill provides for regular examination in classes V and
VIII, and if a child fails, the amendment bill grants a
provision to give her or him additional opportunity to take a
re-examination within two months. Such children will be
provided with two-month remedial teaching to perform
better in the re-examinations. If the students still do not
pass the exam, the state government may decide to detain
them.

Why this question is important?

The TSR Subramanian committee for formulation of the National


Policy on Education has also suggested that ‘no detention’ policy
should be discontinued after Class V. It had recommended
restoration of detention provision, remedial coaching and two
extra chances to each student such to move to a higher class.

79. With regard to Petroleum Conservation Research


Association (PCRA), consider the following statements

1. It is a statutory body.
2. It aims at making oil conservation a national movement.
3. It works under the aegis of Ministry of Petroleum & Natural
Gas, Government of India.
100

Which among the above statements is/are correct?


Page
(a) 1 and 2 only
(b) 2 and 3 only
(c) 1 and 3 only
(d) 1, 2 and 3
Ans: (b) 2 and 3 only

Learning Zone:

● Petroleum Conservation Research Association (PCRA) is

a registered society set up under the aegis of Ministry of


Petroleum & Natural Gas, Government of India.
● As a non-profit organization, PCRA is a national
government agency engaged in promoting energy efficiency
in various sectors of the economy.
● PCRA aims at making oil conservation a national
movement. As part of its mandate, PCRA is entrusted with
the task of creating awareness amongst the masses about
the importance, methods and benefits of conserving
petroleum products & emission reduction.

Why this question is important?

Petroleum Conservation Research Association (PCRA) recently


held Saksham (SanrakshanKshamtaMahotsav) an annual
flagship event.
101

80. With regard to EXIM bank, consider the following


statements
Page
1. It was established in 1982 under the Export-Import Bank of
India Act.
2. It provides direct financial assistance to exporters of plant,
machinery and related service.
3. It provides rediscount of export bills for a period not
exceeding 90 days against short-term usance export bills
discounted by commercial banks.

Which among the above statements is/are correct?

(a) 1 and 2 only


(b) 2 and 3 only
(c) 1 and 3 only
(d) 1, 2 and 3

Ans: (d) 1, 2 and 3

Learning Zone:

Functions of EXIM bank

● It provides direct financial assistance to exporters of plant,


machinery and related service in the form of medium-term
credit.
● Underwriting the issue of shares, stocks, bonds, debentures
of any company engaged in exports.
102
Page
● It provides rediscount of export bills for a period not
exceeding 90 days against short-term usance export bills
discounted by commercial banks.
● The bank gives overseas buyers a credit to foreign importers
for import of Indian capital goods and related services.
● Developing and financing export-oriented industries.

Why this question is important?

Cabinet approves ₹6,000 crore capital infusion in Exim Bank and


an increase in the Exim Bank authorised capital from ₹10,000
crore to ₹20,000 crore.

81. With regard to UNNATI Programme, consider the


following statements

1. It is a capacity building programme on nanosatellite


development.
2. It is an Initiative by the Ministry of Science and technology.

Which among the above statements is/are correct?

(a) 1 only
(b) 2 only
(c) Both 1 and 2
(d) Neither 1 nor 2
103
Page

Ans: (a) 1 only


Learning Zone: The UNNATI Program is to commemorate the
50th anniversary of the first United Nations Conference on the
Exploration and Peaceful Uses of Outer Space (UNISPACE+50).

UNNATI program is planned to be conducted by U R Rao


Satellite Centre (URSC) of ISRO for 3 years in 3 batches and will
target to benefit 90 officials from 45 countries.

Why this question is important?

It is launched by the National space agency and the Indian Space


Research Organisation (ISRO) at the U R Rao Satellite Centre,
Bengaluru.

82.ASER report, often seen in news is related to:

(a) NPA status of Bank


(b) Education
(c) Gene mutation
(d) Infrastructure projects status

Ans: (b) Education


104

Learning Zone: Annual Status of Education Report (ASER) 2018


Page

is published by education non-profit Pratham.


Why this question is important?

The
he survey shows the prevalence of learning deficit and the
poverty of basic reading and arithmetic skills among students in
Indian schools.

105
Page
83. Global Risk Report is published by which among the
following?

(a) World Economic Forum


(b) UNISDR
(c) World Bank
(d) IMF

Ans: (a) World Economic Forum

Learning Zone: The Global Risks Report 2019 has been released
by the World Economic Forum (WEF).

Based on the work of the Global Risk Network, the report


describes changes occurring in the global risks landscape from
year to year and identifies global catastrophic risks.

Why this question is important?

The report explores the interconnectedness of risks and is


intended to raise awareness about the need for a multi-
stakeholder approach to the mitigation of global risk.

84. With regard to Small Farmers’ Agri-Business Consortium


(SFAC), consider the following statements
106

1. It is a society promoted by the Ministry of Agriculture and


Farmers Welfare.
Page
2. It implements the National Agriculture Market (e-NAM)
Electronic Trading platform.
3. It promotes the development of Food processing industries.

Which among the above statements is/are correct?

(a) 1 and 2 only


(b) 2 and 3 only
(c) 1 and 3 only
(d) 1, 2 and 3

Ans: (a) 1 and 2 only

Learning Zone: The Government established Small Farmers’


Agri-Business Consortium (SFAC) as a Society in January 1994
to facilitate agri-business ventures by catalysing private
investment through Venture Capital Assistance (VCA) Scheme in
close association with financial institutions. The role of State
SFACs is to aggressively promote agribusiness project
development in their respective States.

The main functions of SFAC are:

● Promotion of development of small agribusiness through


VCA scheme.
● Helping the formation and growth of Farmer Producer
Organizations (FPOs) / Farmer Producer Companies (FPCs).
● Improving the availability of working capital and
107

development of business activities of FPOs/FPCs through


Equity Grant and Credit Guarantee Fund Scheme.
Page
● Implementation of National Agriculture Market (e-NAM)
Electronic Trading platform.

Why this question is important?

Silver Jubilee Celebrations of the Small Farmers’ Agri-Business


Consortium (SFAC) in 2019.

85. With regard to PradhanMantriRozgarProtsahanYojana


(PMRPY), consider the following statements

1. It is implemented by the Ministry of Labour and


Employment.
2. The scheme is targeted for workers earning wages uptoRs.
15,000/- per month.

Which among the above statements is/are correct?

(a) 1 only
(b) 2 only
(c) Both 1 and 2
(d) Neither 1 nor 2

Ans: (c) Both 1 and 2

Learning Zone:
108
Page
● The scheme
“PradhanMantriRojgarProtsahanYojana”(PMRPY) was
announced in the Budget for 2016-17.
● The objective of the scheme is to promote employment
generation.
● The scheme is being implemented by the Ministry of Labour
and Employment.
● Under the scheme, employers would be provided with an
incentive for enhancing employment by reimbursement of
the 8.33% EPS contribution made by the employer in
respect of new employment.
● The PMRPY scheme is targeted for workers earning wages
uptoRs. 15,000/- per month.

Why this question is important?

The PradhanMantriRozgarProtsahanYojana (PMRPY), the


flagship scheme of the Union Government for employment
generation, crossed the milestone of one crore beneficiaries on
January 14, 2019.

86. With regard to SCATFORM project, recently seen in the


news, consider the following statements

1. It is jointly funded by the Japan International Cooperation


109

Agency (JICA) and Government of India.


Page
2. It aims to improve the quality of forest in the catchment
area by sustainable forest management, soil and moisture
conservation and livelihood development.
3. Andhra Pradesh has recently launched this project.

Which among the above statements is/are correct?

(a) 1 and 2 only


(b) 2 and 3 only
(c) 3 and 4 only
(d) 1, 2 and 3

Ans: (a) 1 and 2 only

Learning Zone: Sustainable Catchment Forest Management


(SCATFORM) Project

 The project is jointly funded by the Japan International


Cooperation Agency (JICA) and Government of India.
 It aims to improve the quality of forest in the catchment
area by sustainable forest management, soil and moisture
conservation and livelihood development.
 The Tripura Forest Department (TFD) is the Executing
Agency of the Project.
 Eighty per cent of the project would be funded by JICA
while Government of India would fund 20 per cent of the
110

project value.
Page
Why this question is important?

Tripura has launched the Sustainable Catchment Forest


Management (SCATFORM) project recently.

87. Which among the following best describes the term ‘Data
Localization’?

(a) The act of storing information held by forest dwellers.


(b) The act of storing data on any device that is physically
present within the borders of a specific country where the
data was generated.
(c) The act of storing data on any device that is not physically
present within the borders of a specific country where the
data was generated.
(d) The act of storing data on a device that is indigenously
made by a country where the data was generated.

Ans: (b) the act of storing data on any device that is


physically present within the borders of a specific country
where the data was generated.

Learning Zone: Data localization is the act of storing data on any


device that is physically present within the borders of a specific
country where the data was generated.
111

Why this question is important?


Page
Data localization is a sensitive issue the world over and more so
in India, given that this is a country of 1.3 billion people with
over 1 billion mobile users. With technology developing rapidly,
more and more devices becoming smarter and the Internet of
Things taking over, a genuine concern around leakage of private
data has gained ground.

88. With regard to Great Indian Bustard, consider the


following statements

1. It is a critically endangered bird.


2. Project Great Indian Bustard is an initiative launched by the
central government to protect them.

Which among the above statements is/are correct?

(a) 1 only
(b) 2 only
(c) Both 1 and 2
(d) Neither 1 nor 2

Ans: (a) 1 only

Learning Zone: Great Indian Bustard is listed in Schedule I of


the Indian Wildlife (Protection) Act, 1972, in the CMS Convention
and in Appendix I of CITES, as Critically Endangered on the
IUCN Red List.
112

It has also been identified as one of the species for the recovery
program under the Integrated Development of Wildlife Habitats of
Page

the Ministry of Environment and Forests, Government of India.


Project Great Indian Bustard — state of Rajasthan —
identifying and fencing off bustard breeding grounds in existing
protected areas as well as provide secure breeding enclosures in
areas outside protected areas

Why this question is important?

Wildlife activists have called for enforcement of recovery plan for


the country’s most critically endangered bird- Great Indian
Bustard. Once the contender for becoming India’s national bird,
the Great Indian Bustard is now facing extinction.

89.‘The Smart Food initiative’ is launched by which among


the following?

(a) FAO
(b) UNICEF
(c) International Crops Research Institute for the Semi-Arid-
Tropics (ICRISAT)
(d) International Rice Research Institute (IRRI)

Ans: (c) International Crops Research Institute for the Semi-


Arid-Tropics (ICRISAT)

Learning Zone: The Smart Food initiative is founded by the


International Crops Research Institute for the Semi-Arid-Tropics
(ICRISAT) and aims to build food systems where the food is good
113

for you (highly nutritious), good for the planet and good for the
Page
smallholder farmer. It is an initiative which will initially focus on
popularizing millets and sorghum.

Why this question is important?

Associations including the Asia-Pacific Association of Agricultural


Research Institutions (APAARI), Forum for Agricultural Research
in Africa (FARA), West and Central African Council for
Agricultural Research and Development (CORAF), Food
Agriculture and Natural Resources Policy Analysis Network
(FANRPAN), and the International Crops Research Institute for
the Semi-Arid Tropics (ICRISAT) together have formed the Smart
Food Executive Council.

90. With regard to Alliance to End Plastic Waste (AEPW),


consider the following statements

1. It is an initiative by G20.
2. Its aim is to develop solutions to mitigate plastic pollution
and promote a circular economy by utilising used plastics.
3. Participants of this alliance are member countries of G20.
Which among the above statements is/are correct?

(a) 1 and 2 only


(b) 2 only
(c) 3 only
(d) 2 and 3 only
Ans: (b) 2 only
114

Learning Zone: The Alliance to End Plastic Waste (AEPW),


comprising about 30 companies, pledged over $1 billion to
Page
eliminate plastic waste across the world. They aim to invest $1.5
billion over the next five years for the same.

The alliance is designed as a non-profit organization. It includes


companies from across North and South America, Europe, Asia,
Southeast Asia, Africa as well as the Middle East is part of the
Alliance.

The aim is to develop solutions to mitigate plastic pollution and


promote a circular economy by utilising used plastics.

Why this question is important?

Plastic waste management is a complex and serious global


challenge that calls for swift action and strong leadership. The
issue of plastic waste is seen and felt all over the world. It must
be addressed. This new alliance is the most comprehensive effort
to date to end plastic waste in the environment

91. Global Economy Watch report, is launched by which


among the following?

(a) World Economic Forum


(b) World Bank
(c) PricewaterhouseCoopers
(d) IMF

Ans: (c) PricewaterhouseCoopers


115

Learning Zone: Global Economy Watch report has been released


by London based multinational professional services network-
Page

PricewaterhouseCoopers (PwC).
PwC’s Global Economy Watch is a short publication that looks at
the trends and issues affecting the global economy and details its
latest projections for the world’s leading economies.

Why this question is important?

As per the report, India should return to a healthy growth rate of


7.6% in 2019-20, if there are no major headwinds in the global
economy such as enhanced trade tensions or supply-side shocks
in oil.

The growth will be supported through the further realisation of


efficiency gains from the newly adopted GST and policy impetus
expected in the first year of a new government.

92. With regard to Triple-Drug Therapy, consider the


following statements

1. It is recommended by WHO.
2. It is used in the treatment of Cancer.
3. India is using this therapy for the elimination of lymphatic
filariasis.

Which among the above statements is/are correct?


116

(a) 1 and 2 only


(b) 2 and 3 only
Page
(c) 1 and 3 only
(d) 1, 2 and 3

Ans: (c) 1 and 3 only

Learning Zone:

Triple drug therapy: The World Health Organization (WHO) is


recommending three drug treatment to accelerate the global
elimination of lymphatic filariasis.

The treatment, known as IDA, involves a combination of


ivermectin,
diethylcarbamazine citrate and albendazole. It is being
recommended annually in settings where its use is expected to
have the greatest impact.
The third drug is used in this therapy will help control adult
worms of lymphatic filariasis. Micro filariasis, which is produced
by adult worms, is the cause of swollen leg. Previously the adult
worms were sterilized by drugs and remained inactive for a year.
Now that period will increase to two years.

The plan is to administer these drugs for two consecutive years.


The life of the adult worm is hardly four years, so it would die a
natural death without causing any harm to the person.

Why this question is important?

A pilot project to administer triple-drug therapy with the long


term aim of eradicating lymphatic filariasis was recently
117

launched in Nagpur, Maharashtra. Nagpur is one of the five


districts in the country and only one in Maharashtra where this
Page

triple-drug therapy campaign is being launched.


93. Project ReWeave, an initiative to revive the handloom
weaving ecosystem in India, has been launched by?

(a) Ministry of Textiles


(b) Google
(c) Microsoft
(d) Ministry of Micro Small and Medium Enterprises

Ans: (c) Microsoft

Learning Zone:
About Project ReWeave: Initiated by Microsoft India (R&D) Pvt.
Limited in 2016 as part of its Philanthropies efforts.

Aim: To revive the handloom weaving ecosystem in India.

Features: Under the initiative, Microsoft is working closely with


NGO partner, Chaitanya Bharathi to provide infrastructure,
financing and marketing support to help weaver families keep
their weaving traditions alive by sustaining livelihoods.

Why this question is important?

Microsoft India has recently launched a new e-commerce


platform ‘re-weave.in’ under project ReWeave to help handloom
weavers.
118
Page
94. Which among the following is/are members of the Arab
League?

1. Bahrain
2. Sudan
3. Libya
4. Syria
5. Jordan

Select the correct answer using the codes given below:

(a) 1, 4 and 5 only


(b) 1, 2 and 4 only
(c) 2 and 5 only
(d) 1, 2, 3, 4 and 5

Ans: (d) 1, 2, 3, 4 and 5

Learning Zone: The Arab League is a regional organization of


Arab countries in and around North Africa, the Horn of Africa
and Arabia.

It was formed in Cairo on 22 March 1945 with six members: the


Kingdom of Egypt, Kingdom of Iraq, Jordan, Lebanon, Saudi
Arabia, and Syria. Currently, the League has 22 members, but
Syria’s participation has been suspended since November 2011,
as a consequence of government repression during the Syrian
119

Civil War.
Page

Why this question is important?


Arab leaders and officials have urged the international
community to support nations hosting Syrian refugees and take
steps to minimise the impact of the refugee crisis. The call was
made in the Beirut Declaration at the Arab Economic and Social
Development Summit that concluded in the Lebanese capital
recently.

95. Which among the following is/are possible benefits of


Methanol blending in Petrol?

1. It can improve air quality.


2. It improves vehicle efficiency.

Select the correct answer using the codes given below:

(a) 1 only
(b) 2 only
(c) Both 1 and 2
(d) Neither 1 nor 2

Ans: (c) Both 1 and 2

Learning Zone: Methanol can be used as an energy producing


fuel, transportation fuel and cooking fuel, cutting down India’s oil
import bill by an estimated 20% over the next few years. Unlike
CNG, using methanol as a transportation fuel would require
minimal alteration in the vehicles.
120

Methanol is a clean-burning fuel that produces fewer smog-


Page

causing emissions — such as sulphur oxides (SOx), nitrogen


oxides (NOx) and particulate matter — and can improve air
quality and related human health issues.

Methanol is most commonly produced on a commercial scale


from natural gas. It can also be produced from renewable sources
such as biomass and recycled carbon dioxide.

As a high-octane vehicle fuel, methanol offers excellent


acceleration and power. It also improves vehicle efficiency.

Why this question is important?

A study, in which methanol (M-15) was blended with petrol and


used in the existing BS-IV standard cars, found that the carbon
dioxide emission had reduced substantially. The study,
conducted by Pune-based Automotive Research Association of
India (ARAI), also found that hydrocarbon emission reduced
marginally, with a slight increase in nitrogen oxides.

96. Flamingo Festival, is held every year in which among the


following place?

(a) Dal Lake


(b) Chilika Lake
(c) Pulicat Lake
(d) Nagaland

Ans: (c) Pulicat Lake


121

Learning Zone: Flamingo Festival is held every year to promote


tourism in Pulicat and Nellapattu. Flamingo Festival is being
Page

organised for the past 12 years.


About Nelapattu Bird Sanctuary: It is considered one of the
biggest habitat for some hundreds of pelicans and other birds.
Located about 20 km north of the Pulicat Lake on the Andhra
Pradesh-Tamil Nadu border, the sanctuary is spread in about
459 hectares.

About Pulicatlake: It is the second largest brackish water lake or


lagoon in India, after Chilika Lake. It straddles the border of
Andhra Pradesh and Tamil Nadu states with over 96% of it in
Andhra Pradesh. The lake encompasses the Pulicat Lake Bird
Sanctuary. The barrier island of Sriharikota separates the lake
from the Bay of Bengal and is home to the SatishDhawan Space
Centre.

97. With regard to ‘Size India Project’, consider the following


statements

1. It will help create an India-specific size chart for the textiles


and garment industry.
2. It is an initiative by the Clothing Manufacturers Association
of India (CMAI).

Which among the above statements is/are correct?

(a) 1 only
(b) 2 only
(c) Both 1 and 2
(d) Neither 1 nor 2
122

Ans: (c) Both 1 and 2


Page
Learning Zone: The ‘Size India’ project will help create an India-
specific size chart for the textiles and garment industry.

Aim: To arrive at standard Indian sizes for apparels.

Significance: The project will reduce overall prices and the


consumers will stand to benefit from it.

Why this question is important?

The Clothing Manufacturers Association of India (CMAI) will work


with the Union Ministry of Textiles in the “Size India” project,
which is expected to be launched March 2019.

98.‘Future of Work’ initiative recently seen in news is


launched by which among the following?

(a) World Economic Forum


(b) International Labour Organization
(c) NITI Aayog
(d) World Trade Organization

Ans: (b) International Labour Organization

Learning Zone: The world of work is undergoing a major process


of change. In order to understand and to respond effectively to
these new challenges the International Labour Organization has
launched a "Future of Work initiative" in order to be able to
123

advance its mandate for social justice.


Page
Why this question is important?

The Global Commission on the Future of Work released its report


on Jan 22. The document calls on governments to take steps to
address the challenges caused by unprecedented transformations
going on in the world of work.

99. With regard to National Girl Child Day (NGCD), consider


the following statements

1. It observed the anniversary of BetiBachaoBetiPadhao


(BBBP) Scheme
2. It aims to generate awareness on the issue of declining
Child Sex Ratio (CSR)
3. Kerala awarded for its good performance on
BetiBachaoBetiPadhao by the central government
Which among the above statements is/are correct?

(a) 1 and 2 only


(b) 2 and 3 only
(c) 1 and 3 only
(d) 1, 2 and 3

Ans: (a) 1 and 2 only

Learning Zone: National Girl Child Day (NGCD) was observed


on 24th January with objectives of generating awareness on the
124

issue of declining Child Sex Ratio (CSR) and create a positive


environment around valuing the girl child.
Page
Why this question is important?

The programme observed the anniversary of


BetiBachaoBetiPadhao (BBBP) Scheme. National Girl Child Day
was first initiated in 2008.

Haryana would be awarded for its good performance for the


'BetiBachaoBetiPadhao' initiative of the national government.

Rajasthan would also be included in the award for its improved


performance for the same.

Andhra Pradesh would be celebrating a weeklong event on the


occasion of National Girl Child Day. The event will include
awareness programs, self-defence training, drawing and painting
competitions for children, child literacy etc.

100. With regard to Autonomous District Council, consider


the following statements

1. It can be formed in any scheduled areas.


2. It has legislative and judicial powers.
3. Governor can modify or divide the boundaries of the areas
under the council by notification.

Which among the above statements is/are correct?

(a) 1 and 2 only


125

(b) 2 and 3 only


(c) 1 and 3 only
Page
(d) 1, 2 and 3

Ans: (b) 2 and 3 only

Learning Zone: As per the Sixth Schedule, the four states viz.
Assam, Meghalaya, Tripura and Mizoram contain the Tribal
Areas which are technically different from the Scheduled Areas.

Though these areas fall within the executive authority of the


state, provision has been made for the creation of the District
Councils and regional councils for the exercise of the certain
legislative and judicial powers.

Each district is an autonomous district and the Governor can


modify/divide the boundaries of the said Tribal areas by
notification.

Why this question is important?

The Union Cabinet has approved a constitutional amendment to


increase the financial and executive powers of the 10
autonomous councils in the Sixth Schedule areas of the
northeast.

126
Page
Road Map to
CLEAR IAS
START

2 1

Read NCERT Text Join ClearIAS


Books from Class Prelims Online Mock
6-12. Test Series

3 4

Learn ClearIAS Notes Read the standard


(Easy-to-Learn reference text books
Online Study Materials) (clearias.com/ias-books)

6 5

Take ClearIAS Mains Finish your optional subject


Mock Exams and and GS Topics as per
polish your answer ClearIAS Prelims cum Mains
writing skills Integrated approach.

Get your bio-data Success!


reviewed by ClearIAS
Experts.Take ClearIAS
Mock Interviews.

Clear IAS by Self-Study and Online Preparation


www.clearias.com
ClearIAS Prelims
Online MockTest
Series

The Test Series You


Should NOT Miss!
Join now!

www.clearias.com

S-ar putea să vă placă și